Está en la página 1de 56

@CHOLINOMIMETIC

1. Which one of the following statements concerning the parasympathetic


nervous system is correct?
1.The parasympathetic system uses
norepinephrine as a neurotransmitter.
2.The parasympathetic system often discharges as a single,
functional system.
*3.The parasympathetic division is involved in accommodation
of near vision, movement of food, and urination.
4.The postganglionic fibers of the parasympathetic division are
long compared to those of the sympathetic nervous system.
5.The parasympathetic system controls the
secretion of the adrenal medulla.
2. Which one of the following is characteristic of
parasympathetic stimulation?
1.Decrease in intestinal motility.
2.Inhibition of bronchial secretion.
*3.Contraction of sphincter muscle in the iris of the eye (miosis).
4.Contraction of sphincter of urinary bladder.
5.Increase in heart rate.
3. A patient with an acute attack of glaucoma is treated with
pilocarpine. The primary reason for its effectiveness in this condition is:
1.action to terminate acetylcholinesterase.
2.selectivity for nicotinic receptors.
3.ability to inhibit secretions, such as tears, saliva, and sweat.
*4.ability to lower intraocular pressure.
5.inability to enter the brain.
4. A soldier's unit has come under attack with a nerve agent.
The symptoms exhibited are skeletal muscle paralysis, profuse
bronchial secretions, miosis, bradycardia, and convulsions.
The alarm indicates exposure to an organophosphate. What is the
correct treatment?
1.Do nothing until you can confirm the nature of the nerve agent.
2.Administer atropine, and attempt to confirm the nature of the nerve agent
.
3.Administer atropine.
*4.Administer alloximum.
5. A myasthenia gravis patient would be expected to have improved
neuromuscular function after being treated with:
1.Carbacholinum
*2.Proserinum
3.Atropinum
4.Pilocarpinum
5.Physostigminum
6. The drug of choice for treating decreased salivation is:
1.physostigmine.
2.scopolamine.
3.carbachol.
4.acetylcholine.
*5.pilocarpine.
7. Which of the following profile is correct for Pilocarpine?
*1.Is used to lower intraocular pressure in glaucoma
2.Is cleaved by acetylcholinesterase
3.Selectively binds to nicotinic receptors
4.Inhibits acetylcholinesterase
5.Inhibits secretion of gastrointestinal glands
8. Which of the following profile is correct for Neostigmine (Proserinum)?
1.Is contraindicated in glaucoma
2.Is cleaved by acetylcholinesterase
3.Selectively binds to nicotinic receptors
*4.May result in bowel hypermotility, salivation and sweating
5.Exacerbates tubocurarine poisoning
9. A 50-year old male with symptoms of phosphoorganic poisoning
was taken to emergency department. Which of the following drugs
should be used as a drug of first aid?
1.Unithiolum
2.Proserinum
*3.Alloximum
4.Arminum
5.Carbacholinum
10. A doctor has decided to include anticholinesterase drug
in improving mental development of child with cerebral palsy.
Which of the following drug is it?
1.Proserinum
2.Arminum
3.Carbacholinum
*4.Galanthamini hydrobromidum
5.Physostigmini salicylas
11. A 60-year old male patient was treated with cholinomimetic
to restore movement of the paralyzed extremities. Which of the
following drug was used for this purpose?
*1.Proserinum
2.Arminum
3.Carbacholinum
4.Aceclidinum
5.Physostigmini salicylas
12. Indicate the drug for treatment of glaucoma:
1.Kalymin
2.Dipiroximum
*3.Pilocarpinum
4.Galanthamini hydrobromidum
5.Alloximum
13. Applications of this group of drug are myasthenia,
glaucoma, intestinal and urinary bladder atony.
Determine group of the drugs.
1.M-cholinomimetics
2.N-cholinomimetics
*3.Cholinesterase inhibitors
4.Cholinesterase reactivators
14. Indicate the main features of lobeline action.
1.Directly stimulates the respiratory center
2.Causes the relaxation of bronchial smooth muscles
*3.Acts by reflex stimulation of carotide zone receptors
4.Stimulates respiratory muscles
15. What is the mechanism of the increase of blood pressure by nicotine?
1.Direct stimulation of vascular adrenoreceptors
2.Stimulation of cardiomyocytes receptors
3.Direct influence on the vascular wall
*4.Stimulation of the ganglion receptors
5.Stimulation of the renin secretion
@Chol-blok
1.A 75-year-old man who was a smoker is diagnosed with
chronic obstructive pulmonary disease and suffers from
occasional bronchospasm. Which of the following would be
effective in treating him?
*1.Ipratoprium
2.Scopolamine
3.Hygronium
4.Extr. Belladonnae spissi
5.Benzohexonium
2.Which of the following may precipitate an attack
of open-angle glaucoma applied on the eye?
1.Physostigmine
*2.Atropini sulfas
3.Pilocarpine
4.Dithylinum
5.Platyphyllini hydrotartras
3.The prolonged apnea sometimes seen in patients who had undergone
an operation in which succinylcholine was employed as a
muscle relaxant has been shown to be due to:
1.urinary atony
*2.depressed levels of plasma cholinesterase
3.mutation in acetylcholinesterase
4.mutation in the nicotinic receptor at the
neuromuscular junction
4.A 50-year-old male farm worker is brought to the emergency room.
He was found confused in the orchard and since then has
lost consciousness.His heart rate is 45 and his blood pressure
is 80/40 mm Hg. He is sweating and salivating profusely.
Which of the following treatments is indicated?
1.Physostigmine
2.Pentaminum
3.Mellictinum
*4.Atropine
5.Hygronium
5.M-cholinoblockers may cause all the following effects,
excepts:
1.Tachycardia
2.Dry mouth
3.Blurred vision
*4.A-V block
5.Constipation
6.In a case of Belladonna poisoning proserine antagonizes
the following symptoms, except:
1.Palpitation
2.Dry mouth
3.Blurred vision
*4.Hallucination
5.Urinary retention
7.Indicate the drug for treatment of renal and biliary colic:
*1.Atropine
2.Hygronium
3.Pilocarpine
4.Tubocurarine
5.Dithyline
8.Contraindication for use of this group of drugs is glaucoma.
Determine the group of drugs.
1.M-cholinomimetics
2.N-cholinomimetics
3.Cholinesterase inhibitors
4.Myorelaxants
*5.M-cholinoblockers
9.Indicate the drug of the choice for antidote therapy
of Belladonna poisoning
1.Proserinum
2.Arminum
3.Carbacholinum
4.Aceclidinum
*5.Physostigmini salicylas
10.The skeletal muscles that are most sensitive
to the action of tubocurarine are the:
1.Muscles of the trunk
*2.Muscles of the arms and legs
3.Respiratory muscles
4.Muscles of the head, neck, and face
5.Abdominal muscles
11.Determine the group of drugs for skeletal muscles relaxation.
1.M-cholinomimetics
2.N-cholinomimetics
3.Cholinesterase inhibitors
*4.N-cholinoblockers
5.M-cholinoblockers
12.Determine the most preferable drug among M-cholinoblockers
for the treatment of bronchial asthma:
1.Atropine
2.Scopolamine
*3.Ipratropium
4.Methacinum
5.Plathyphilline
13.Administration of tubocurarine during surgery can enhance
the development of ðostoperative paralytic ileus.
What is the mechanism of this side effect?
1.Inhibition of M- cholinoreceptors
2.Inhibition of cholinesterase
*3.Ganglion blocking activity
4.Stimulation of M-cholinoreceptors
5.Stimulation of M and N-cholinoreceptors
14.A patient with a fracture of the hand was taken
to the emergency room. To perform reposition of bones
it was necessary to relax skeletal muscles.
Which of the following drugs was administered?
1.Plathyphilline
2.Scopolamine
*3.Tubocurarine
4.Succinylcholine
5.Neostigmine
15.To mark a drug with sedative influence on CNS:
1.Tubocurarine
2.Atropine
*3.Scopolamine
4.Metacinum
5.Penthaminum
@Adrenomimetics
1. A 60-year-old male with congestive heart failure (CHF) is treated with dobuta
mine.
Select the mechanism of action of dobutamine.
1.alfa-adrenergic agonist
2.alfa-adrenergic antagonist
*3.beta-adrenergic agonist
4.beta-adrenergic antagonist
5.Mixed alfa and beta-agonist
2. The drug of choice for the treatment of anaphylactic shock is:
*1.Epinephrinum
2.Noradrenalinum
3.Isadrinum
4.Dobutaminum
5.Atropinum
3. Which of the following agents will increase pulse pressure?
1.Metoprolol
2.Dopamine
3.Noradrenalinum
*4.Epinephrinum
5.Salbutamolum
4. Which of the following agents might mask the hypoglycemia in treated diabetic
s?
1.alfa-adrenergic agonist
2.alfa-adrenergic antagonist
*3.beta-adrenergic agonist
4.beta-adrenergic antagonist
5.Cholinergic agonist
5. Epinephrine may be mixed with certain anesthetics, such as procaine, in order
to
1.Stimulate local wound repair
2.Promote hemostasis
3.Enhance their interaction with neural membranes and their
ability to depress nerve conduction
*4.Retard their systemic absorption
5.Facilitate their distribution along nerves and fascial planes
6. A 16-year-old male treated for bronchial asthma develops skeletal muscle trem
ors.
Which of the following agents may be responsible for this finding?
1. Ipratropium
2. Atropinum
3. Beclomethasone
4. Cromolyn
*5.Salmeterol
7. A patient with bronchial asthma addresses his doctor with complaints about
unpleasant palpitations that occur after usage of
inhalation form of isoprenaline (isadrinum).
What is the cause of this side effect?
1.Stimulation of alfa1-adrenoceptros
2.Stimulation of alfa2-adrenoceptros
*3.Stimulation of beta1-adrenoceptros
4.Stimulation of beta2-adrenoceptros
5.Inhibition of m-chlolinoceptors
8. Which of the following bronchodilators work through cholinergic receptor anta
gonism?
1.Salbutamol
2.Cromolyn
3.Ephedrine
*4.Ipratropium
5.Salmeterol
9. A 68-year-old man presents to the emergency department with acute heart failu
re.
You decide that this patient requires immediate drug therapy
to improve his cardiac function. Which one of the following drugs
would be most beneficial?
1.Salbutamol
*2.Dobutamine
3.Epinephrine
4.Norepinephrine
5.Phenylephrine
10. Remedies for nasal stuffiness often contain which one of the following drugs
?
1.Salbutamol
2.Atropine
3.Epinephrine
4.Norepinephrine
*5.Naphthisin
11. Which one of the following drugs, when administered intravenously,
can decrease blood flow to the skin, increase blood flow to skeletal muscle,
and increase the force and rate of cardiac contraction?
*1.Epinephrine
2.Isadrin
3.Norepinephrine
4.Phenylephrine
5.Terbutaline
12. How you can explain to the patient rapid reduction in ephedrine`s action aft
er
giving the second dose of the drug?
1.Potentiation
2.Tolerance
3.Cumulation
4.Sensibilisation
*5.Tachyphylaxis
13. Major effects mediated by alfa1-adrenoreceptors are following, EXEPT:
1.Increased peripheral resistance
*2.Vasodilation
3.Increased blood preasure
4.Mydriasis
5.Increased closure of internal sphincter of the bladder
14. Major effects mediated by beta2-adrenoreceptors are following, EXEPT:
*1.Vasoconstriction
2.Slightly decreased periferal resistance
3.Bronchodilation
4.Increased release of glucagon
5.Increased of glycogenolysis
15. Following drugs are Suitable for treatment of broncial astma EXEPT:
1.Salmeterol
2.Ephedrine
3.Epinephrine
*4.Norepinephrine
5.Terbutaline
@ADRENOBLOCKERS, SYMPATHOLYTICS
1.Both phentolamine and prazosin
*1.Are competitive antagonists of alfa-adrenergic receptors
2.Have potent vasodilator action on vascular smooth muscle
3.Enhance gastric acid secretion through histamine-like effect
4. Cause hypotension and bradycardia
5. Are used chronically for the treatment of primary hypotension
2.A 58-year-old male with angina is treated with atenolol.
Select the mechanism of action of atenolol.
1.alfa-adrenergic agonist
2.alfa-adrenergic antagonist
3.beta-adrenergic agonist
*4.beta-adrenergic antagonist
5.Mixed alfa-and beta-agonist
6.Mixed alfa-and beta-antagonist
3.Which of the following is the side effect of Phentolaminum?
1.Dependence
2.Increased blood pressure
*3.Tachycardia
4.Depression
5.Gastrointestinal ulceration
4.In a hypertensive patient who is taking insulin to treat diabetes,
which of the following drug is to be used with extra caution and
advice to the patient?
1.Hydralazine
2.Prazosin
3.Pirroxan
*4.Propranolol
5.Methyldopa
5.The nonselective beta-adrenergic blocking agent that is
also a competitive antagonist of alfa1-adrenoceptors is:
1.Timolol
2.Nadolol
3.Pindolol
4.Acebutolol
*5.Labetalol
6.A 65-year-old male has a blood pressure
of 170/105 mmHg. Which of the following would be effective
in lowering this patient s blood pressure?
1.Adrenalin
2.Terbutaline
3.Dobutamine
*4.Prazosin
5.Scopalamine
7.Indicate the sympatholytic
*1.Octadine
2.Anaprilinum
3.Prazosin
4.Acebutalol
5.Labetalol
8.beta-Adrenoblockers are used to treat
1.hypotonia
*2.angina pectoris
3.daibetes melitas
4.ulcerative disease
5.bronchial asthma
9.Decrease in vascular tone following reserpine administration
is due to
1.antagonism of noradrenaline action
*2.depletion of noradrenaline store
3.inhibition of MAO
4.inhibition of noradrenaline synthesis
5.direct relaxation of smooth muscle of blood vessels
10.Which of the following statements concerning prazosin
is not correct?
1.It is potent antihypertensive agent
2.It causes vasodilatation
*3.It blocks preferentially alfa-2 adrenoreceptors
4.It causes relatively modest degree of tachycardia.
5.It is effective in the treatment of prostate hypertrophy
11.All the following are the indications for clinical use
of propranolol except
1.hypertension
2.tachyarrhythmia
*3.Raynaud's disease
4.angina pectoris
5.myocardial infarction
12.alfa-Adrenoceptor blockers may cause the following side effects
1.stimulates CNS
*2.reflex tachycardia
3.periferal vasoconstriction
4.mydriasis
5.constipation
13.Propranolol is effective in the treatment of hypertension due to
1.increase in peripheral vascular tone
*2.decrease of cardiac output
3.inhibition of NA release from presynaptic membrane
4.activated of renin release
5.action on CNS
14. A 38-year-old male has resently started monotherapy for mild
hypertension. At his most recent office visit, he complains of
tiredness and not beign able to complete three sets of tennis.
Which one of the following drugs is he most
likely to be taking for hypertension?
1.Albuterol
*2.Atenolol
3.Ephedrine
4.Phentolamine
5.Prazosin
15.Doctor prescribed selective alfa1-adrenoceptor blocker
for treatment of hypertensive patient. Which of the
following drugs belongs to this group?
1.Metoprolol
2.Amiodarone
3.Tropaphenum
*4.Prazosin
5.Phentolamine
16.A 72-year woman suffering from angina pectoris was
treated by propranolol (anaprilinium).
On the background of the treatment cough with
bronchospastic components was developed.
Doctor replaced propanolol with metoprolol.
This resulted in abolishing of respiratory
disturbances. What is the reason for the
beneficial effect of metoprolol?
1.Block of M-cholinoceptors
*2.Absence of action upon beta2-adrenoceptors
3.Stimulating of M-cholinoceptors
4.Inhibition of M-cholinireceptors
5.Absence of action on alfa-adrenoceptors
@ Analgesics
1. In complex treatment of a patient with angina pectoris non-narcotic analgesic
exerting antiplatelet activity was included. Point out this agent.
*1. Aspirin
2. Morphine
3. Acetaminophen
4. Metamizole
5. Tramadol
2. A 60-year-old male is brought to emergency room. He is comatose and his
pupils are constricted. Physician suspects opium overdose.
What is the best drug to be administered?
1. Flumazanil
*2. Naloxone
3. Sodium bicarbonate
4. Calcium carbonate
5. Atropine
3. Neogalenic drug made of opium contains 50% of morphinum.
It is effective in traumatic and spastic pain.
It causes tolerance and drug addiction. Determine the drug.
1. Analginum
2. Morphini hydrochloridum
*3. Omnoponum
4. Nalorphini hydrochloridum
5. Promedolum
4. Pirasolone derivative: is high water-soluble.
Is administered orally and parentally.
Obtains quick but shot effect in headache, toothache, neuralgia.
Side effects: leukopenia, agranulocytosis. Determine the drug.
1. Paracetamol
2. Morphini hydrochloridum
3. Nalorphini hydrochloridum
*4. Analginum
5. Celecoxibum
5. Which of the following opioid agonists is administered only
by the parenteral route?
1. Morphine
2. Codeine
*3. Fentanyl
4. Methadone
5. Naloxone
6. What effects are not the results of acute poisoning with morphine:
1. A coma
2. A respiratory depression
*3. Hyperthermia
4. Decreased body temperature
5. Narrowed pupils
7. All the following concerning codeine is correct exept:
1. it is an opium alkaloid
2. it is effective antitussive
3. it suppresses respiration
4. it causes constipation
*5. it is potent analgesic
8. A young man is brought into the emergency room. He is unconscious,
and he has pupillary constriction and depressed respiration.
You note needle marks on his legs. You administer naltrexone, and he awakens.
This agent was effective because:
1. the patient was suffering from an overdose by an opioid.
*2. naltrexone antagonizes opiates at the receptor site.
3. naltrexone is a stimulant of the CNS.
4. naltrexone binds to the opioid and inactivates it.
9. A heroin addict has entered a rehabilitation program
that requires she take methadone.
Methadone is effective in this situation because it:
1. is an antagonist at the morphine receptors.
2. has less potent analgesic activity than heroin.
*3. is longer acting than heroin; hence, the withdrawal
is milder than with the latter drug.
4. does not cause constipation.
5. is nonaddictive.
10. Which of the following statements about morphine is correct?
1. It is used therapeutically to relieve pain caused by severe head injury.
2. Its withdrawal symptoms can be relieved by naloxone.
3. It causes diarrhea.
4. It is most effective by oral administration.
*5. It rapidly enters all body tissues,
including the fetus of a pregnant woman.
11. The pain of a patient with bone cancer has been
managed with a morphine pump.
However, he has become tolerant to morphine.
Which of the following might be indicated to ameliorate his pain?
1. Aspirin
2. Codeine
*3. Fentanyl
4. Methadone
5. Analgin
12. To prevent the onset of myocardial infarction in a patient with angina
pectoris aspirin was used. Due to which of listed effects
aspirin is beneficial in this case?
1. Antiinflammatory
*2. Antiplatelet
3. Analgesic
4. Antipyretic
5. Ulcerogenic
13. Non-opioid analgesics may exert the following side-effects except:
1. Damage of gastric mucosa
2. Suppression of haemopoiesis
3. Bronchospasm
*4. Drug dependence
5. Antiplatelet
14. Morphine exerts all the following effects except:
1. euphoria
2. sedation
3. cough suppression
4. miosis
*5. diarrhea
15. Opioid analgesics are not used for:
1. relief of pain associated with cancer
2. acute pulmonary edema
3. myocardial infarction
*4. toothache
5. neuroleptanalgesia
@ANTIDEPRESANTS
1. The most common adverse effect associated
with the tricyclic antidepressants is
*1.Anticholinergic effects
2.Seizures
3.Arrhythmias
4.Hepatotoxicity
5.Nephrotoxicity
2. A 40-year-old male with repetitive obsessive behavior
that prevents him from carrying out simple tasks is treated
with fluoxetine. How is fluoxetine classified?
1.As an MAO inhibitor
2.As a tricyclic nonselective amine reuptake inhibitor
3.As a heterocyclic nonselective amine reuptake inhibitor
*4.As a selective serotonin reuptake inhibitor
5.As an adrenergic receptor inhibitor
3. A 41-year-old female is seen in the psychiatric clinic
for a follow-up appointment. She has been taking an antidepressant
for three weeks with some improvement in mood.
However, she complains of drowsiness, palpitations,dry mouth,
and feeling faint on standing. Which antidepressant is she taking?
*1.Imizinum
2.Carbamazepinum
3.Fluoxetine
4.Lithium salts
5.Nialamidum
4. What is not the clinical use of nootropic drugs?
1.Mental insufficiency
2.Depression
*3.Neurosis
4.Asthenia syndrome
5.Apathy
6.Blood-stroke
5. To mark the actoprotector:
1.Amitriptylinum
2.Pantocrinum
3.Pyracetamum
*4.Bemethylum
5.Imizinum
6. A 55-year-old teacher began to experience changes in mood.
He was losing interest in his work and lacked the
desire to play his daily tennis match. He began to have difficulty
thinking and concentrating and lacked
decisiveness. He was preoccupied with feelings of guilt,
worthlessness, and hopelessness. Physical and laboratory
tests were unremarkable. Which of the following drugs
might be useful in this patient?
*1.Fluoxetine
2.Pyracetamum
3.Caffeine
4.Cavintonum
5.Lithium salts
7. A 25-year-old woman had a long history of depressive
symptoms accompanied by body aches.
Physical and laboratory tests were unremarkable.
Which of the following drugs might be useful in this patient?
1.Sydnocarbum
2.Coffeinum
3.Pyracetamum
4.Lithium salts
*5.Amitriptylinum
8. A 51-year-old woman with symptoms of major depression
also has narrow-angle glaucoma. Which of the following
antidepressants should be avoided in this patient?
*1.Amitriptyline
2.Nialamidum
3.Fluoxetine
4.Lithium salts
5.Cavintonum
9. Indicate the drug which inhibit monoaminoxidase:
1.Imizinum
2.Fluoxetine
3.Amitriptyline
*4.Nialamidum
5.Lithium
10. Caffeine exerts all the following actions EXEPT:
1.Increase heart rate
2.Stimulation of hydrochloric acid production
3.Diuretic action
4.Relax smooth muscles of bronchioles
*5.Decrease blood pressure
11. How you can explain to the patient feelings of fatigue and
sedation after giving withdrawal of caffeine?
*1.Physical dependence
2.Tolerance
3.Cumulation
4.Sensibilisation
5.Tachyphylaxis
12. Mechanism of action of adaptogens include:
1.Activation of cholinergic transmission
2.Activation of adrenergic transmission
3.Blockade of adenosine receptors
*4.Activation of RNA and protein synthesis due to
membrane protective activity
5.Blockade of dopamine receptors
13. Onset of adaptogen`s action is:
1.1-3 days
2.7-10 days
3.2 weeks
4.1 month
*5.3 months
14. Nowadays nicotine has following therapeutic use:
1.hypotension
*2.smoking cessation therapy
3.intestine atonia
4.all of above
5.no therapeutic use
15. Onset of antidepressants action is:
1.7-10 days
2.1-2 weeks
*3.2-12 weeks
4.15 weeks
5.5 months
@NEUROLEPTICS
1. Which of the following is described as a competitive
benzodiazepine receptor antagonist?
1. Ketamine
2. Chlordiazepoxide
*3. Flumazenil
4. Midazolam
5. Triazolam
2. Which neuroleptic is applied for the neuroleptanalgesia:
1. Aminasinum
*2. Droperidolum
3. Diazepamum
4. Haloperidolum
5. Triphtasinum
3. To specify the basic action of neuroleptics:
1. Antiirritant
*2. Antipsychotic
3. Local anesthetic
4. Hypnotic
5. Hypotensive
4. Indicate the neuroleptics:
*1. Aminazinum
2. Lithium carbonate
3. Piroxicam
4. Ketamine
5. Natrii bromidum
5. Indicate the main effects of bromides
1. Anxiolytic effect
2. Improve memory
3. Increase of motor activity
*4. Sedative action
5. Increase mood
6. Indicate the applications for use of lithium
1. Acute manic excitement.
*2. Prevention of manic-depressive psychosis attacks.
3. Vomiting of central origin
4. Neuroleptanalgesia
5. Potentiation of general anesthetics,
hypnotic agents and opioid analgesics
7. Indicate the tranquilizers
1. Aminazinum
2. Haloperidol
*3. Sibazonum
4. Lithium carbonate
5. Natrii bromidum
8. Indicate the characteristic effects of the tranquilizers
from the group of benzodiazepine derivatives.
*1. Anxiolytic effect
2. Increase mood
3. Anticonvulsive action
4. Decrease of tonicity of skeletal muscles
5. Potentiation of general anesthetics,
hypnotic agents and opioid analgesics
9. Haloperidol may be best characterized by which
of the following statements
1. It is classified as a phenothiazine
2. It is a selective d2 receptor agonist
3. Its mechanism of action is completely
different from that of chlorpromazine
*4. It is more potent as an antipsychotic drug
than is chlorpromazine
5. It produces a lower incidence of extrapyramidal reactions
than does chlorpromazine
10. Determine the tranquilizer which doesn t have significant
hypnotic and myorelaxant action:
1. Diazepamum
*2. Gidasepamum
3. Phenazepamum
4. Nitrazepamum
11. Indicate the mechanism of tranquilizing action of sibazonum.
1. Interaction with adrenoceptors
2. Interaction with cholinoceptors
*3. Interaction with benzodiazepine receptors
4. Inhibition of the adrenergic processes in the CNS
5. Stimulation of the dopaminergic processess in the CNS
12.Indicate the side-effects which may be observed in treatment
by tranquilizers exept:
1. Impairment of movement coordination
2. Sleepiness
3. Decrease of tonicity of skeletal muscles
4. Drug dependence
*5. Extrapyramidal disorders
13. Indicate the mechanism of antipsychotic effect of neuroleptics
1. Stimulation of the adrenergic processes in the CNS
*2. Inhibit of the dopaminergic processes in the CNS.
3. Potentate GABA receptors
4. Interaction with benzodiazepine receptors
5. Inhibit of amine reuptake
14. Indicate the applications for use of neuroleptics
*1. Psychoses
2. Parkinsonism disease
3. Neuroses
4. Sleeping disorders
5. Spastic states
15. Indicate the mechanism of action of bromides.
1. Blockade of dopamine receptors
*2. Amplification and concentration of inhibitory processes
in the brain cortex
3. Interaction with adrenoreceptors
4. Arrest of psychomotor excitement
@GENERAL ANESTHETIC
1.Which of the following inhalation anesthetic is most likely
to produce hepatotoxicity?
1.Isoflurane
2.Enflurane
3.Methoxyflurane
*4.Halothane
5.Nitrous oxide
2.A 55-year-old female given a general anesthetic for a surgical
procedure develops hyperthermia, hypertension,
hyperkalemia, tachycardia, muscle rigidity, and
metabolic acidosis. Which of the following general
anesthetics did she receive?
*1.Ketamine
2.Midazolam
3.Thiopental
4.Propofol
5.Halothane
3.Mark the hypnotic from group of benzodiazepines:
1.Phenobarbital
2.Zolpidem
3.Cyclobarbiyalum
*4.Nitrazepam
5.Halothane
4.Special antagonist of benzodiazepines:
*1.Flumazenil
2.Pralidoxime
3.Triazolam
4.Triclophos
5.Zolpidem
5.Mark new nonbenzodiazepine hypnotic:
*1.Zopiclone
2.Chlorali hydras
3.Paraldehyde
4.Promethazine
5.Amitriptyline
6.Indicate the drugs used for inhaleted anesthesia:
*1.Ether
2.Thiopental sodium
3.Ketamin
4.Natrii oxybutiras
5.Propanidid
7.Halogenated anesthetics may produce malignant
hyperthermia in:
1.patients with poor renal function
2.patients allergic to the anesthetic
3.pregnant women
4.alcoholics
*5.patients with a genetic defect and receive succinylcholine
8.Which one of the following is most likely to require
administration of a muscle relaxant?
1.Ethyl ether
2.Halothane
3.Methoxyflurane
4.Benzodiazepines
*5.Nitrous oxide
9.Which one of the following is a potent intravenos
anesthetic but a weak analgesic?
*1.Thiopental
2.Benzodiazepines
3.Ketamine
4.Nitrous oxide
5.Isoflurane
10.Which one of the following is a potent analgesic but
a weak anesthetic?
1.Methoxyflurane
2.Succinylcholine
3.Diazepam
4.Halothane
*5.Nitrous oxide
11.Indicate the drugs used for intravenous anesthesia:
1.Nitrous oxide
2.Enfluran
*3.Propofol
4.Halothane
5.Isoflurane
12.Halothane can't cause the following side effect;
1.depression of the heart contraction
2.decrease of BP
*3.increase in heart rate
4.decrease secretion of glands
5.depression of respiration
13.Nitrouse oxide is characterised by the following
properties exept:
1.good analgesia
2.quick action
3.poor muscle relaxation
*4.high liver toxicity
5.usage in obstetrics and dental surgery
14.Ketamin is characterised by the following
properties exept:
*1.long acting generalanaesthetic
2.increasing cardiac output
3.producing profound analgesia
4.not abolishing reflexes
5.used for short operations
15.Stage I of anaesthesia (analgesia stage) is
characterised by the following exept:
1.abolishing of pain
2.impaired ability to see and hear
3.loss of conscience
*4.relaxation of skeletal muscles
5.disturbances of coordination of movements
16.The III stage of general anaesthesia (stage of surgical
anaesthesia) is manifested by folloming exept:
1.four levels of its development
*2.excitement
3.progressive decrease in muscular tone
4.eye movements cease,pupil is fixed
5.fall in BP
17.Halothane is characterised by the following
properties exept:
1.volatile liquide
2.non-irritant agent
3.non-flammable
4.potent anaesthetic
*5.potent relaxant of skeletal muscles
@ GIT
1. Administration of pirenzepine in patients with peptic ulcer is not
accompanied by numerous side effects, which are characteristic for
atropine and other M-cholinoblckers due to:
1.Inability to penetrate through blood brain barrier
*2.Selective inhibition of M1-cholinoceptors
3.Inhibition of all types of M-cholinoceptors
4.Inhibition of cholinesterase
5.Significant protein binding
2. A young woman with breast cancer after surgery was treated with a
chemotherapeutic complex. At the very beginning of treatment the
patient experienced nausea and vomiting.
Choose the best agent to relieve patient's condition.
1.Hyoscine (Aeronum)
2.Diphenhydramine (Dimedrolum)
3.Scopolamine
4.Chlorpromazine (Aminazinum)
*5.Metoclopramide
3. Patient with gastric ulcer was treated with omeprazole.
This agent is capable of decreasing HCL production because it.
1.Neutralizes gastric acid
2.Blocks H2-histamine receptors
*3.Inhibits the H+, K+-ATPase proton pump
4.Coats gastric mucosa
5.Blocks muscarinic receptors
4. A 45-year-old woman presents to the emergency room with severe
epigastric pain. On patient examination and laboratory findings,
an acute pancreatitis is diagnosed. Which of
the drugs is administered in this case?
1.Pancreatin
2.Mezym forte
*3.Contrycal
4.Pirenzepine
5.Omeprazole
5. A patient suffering from gastric ulcer was treated
with cholinotropic drug, selectively blocking M-cholinoceptors
of parietal cells of gastric glands.
Point out this drug?
1.Atropine
2.Plathyphylline
3.Methacinum
*4.Pirenzepine
5.Hyoscine (Scopolamine)
6. A patient addressed doctor with complaints of heartburn.
On examination ulcerative esophagitis was diagnosed and inhibitor
of proton pump was prescribed. Which of the following drugs
are prescribed in this case?
1.Cimetidine
2.Ranitidine
*3.Omeprazole
4.Misoprostol
5.Metronidazole
7. Patient with peptic ulcer came to the doctor. After examination doctor
prescribed famotidine, aiming at inhibition of gastric acid secretion.
What is the mechanism of this drug action?
1.Inhibition of proton pump
2.Neutralization of gastric acid
3.Prevention of histamine release
4.Blockade of H1-histamine receptors
*5. Blockade of H2-histamine receptors
8. The following mechanism is responsible for the
ulcer-healing effect of bismuth compounds:
1.Acid neutralisation
2.Decrease in gastric secretion
*3.Inhibition of Campylobacter pylori
4.Inhibition of M-cholinoceptor
5.Inhibition of gastroduodenal motility
9. The mechanisms of Bisacodyl includes:
1.Adding bulk to the stool
*2. Increasing peristaltic activity
3.Emulsifying aqueous and fatty substances with stool
4.Lubricating the passage of stool
5.All of the above
10. What drug is used orally in constipation and
intramuscularly in cupping of hypertensive crisis?
1. Unithiolum
2. Lidocaini hydrochloridum
*3. Magnesii sulfas
4. Natrii hydrocarbonas
5. Pirenzepine
11. Which of the following agents interferes with most
of the cytochrome P450 enzymes and, thus,
leads to many drug-drug interactions?
1.Famotidine
2.Omeprazole
*3.Cimetidine
4.Sucralfate
5.Pirenzepine
12. A couple celebrating their fortieth wedding anniversary
is given a trip to Peru. Due to past experiences while traveling,
they ask their doctor to prescribe an agent for diarrhea.
Which of the following would be effective?
1.Omeprazole
*2.Loperamide
3.Famotidine
4.Lorazepam
5.Bisacodyl
13. Indicate the drug which decreases the appetite:
1.Tinctura Absinthii
*2.Chlorpheniramine (Desopimon)
3.Insulin
4.Retabolil
5.Pepsin
14. Indicate drug which stimulates the secretion of bile:
1.Magnesium sulfate
*2.Allocholum
3.Contrycal
4.Metoclopromide
5.Siliborum
15. Indicate drug which promotes the release of bile:
*1.Magnesium sulfate
2.Allocholum
3.Contrycal
4.Metoclopromide
5.Siliborum
@ANTIPARCINSONIC
1. Peripheral adverse effects of levodopa, including nau¬sea,
hypotension, and cardiac arrhythmias, can be diminished
by including which of the following drugs in the therapy?
1.Amantadine
2.Bromocriptine
*3.Carbidopa
4.Cyclodolum
5.Diazepam
2. Modest improvement in the memory of patients with
Alzheimer disease may occur with drugs that increase
transmission at which of the following receptors?
1.Adrenergic
*2.Cholinergic
3.Dopaminergic
4.GABAergic
5.Serotonergic
3. Carbidopa is useful in the treatment of Parkinson s disease because it
*1. Is a precursor of levodopa
2. Is a dopaminergic receptor agonist
3. Prevents peripheral biotransformation of l-dopa
4. Prevents a breakdown of dopamine
5. Promotes a decreased concentration of l-dopa
in the nigrostriatum
4. The preferred treatment of status epilepticus
is intravenous administration of
1. Chlorpromazine
*2. Diazepam
3. Succinylcholine
4. Cyclodolum
5. Ethosuximide
5. Which of the following is a selective inhibitor of
monoamine oxidase type B (MAO-B) and,
therefore, useful in treating Parkinsonism?
1. Bromocriptine
2. Carbidopa
*3. Selegiline
4.Natrii volproats
5.Ethosuximide
6. Which one of the following therapies would be
most appropriate in the myoclonic convulsions?
1.Ethosuximide
2.Carbamazepine
3.Diazepam
4.Levodopa
*5.Natrii volproats
7.A dopamine receptor agonist that is useful
in the therapy of Parkinson s Disease is
1. Selegiline
*2. Bromocriptine
3. Apomorphine
4. Diazepam
5. Belladonna
8.The drugs of first choice for preventtion of
generalsed tonic-clonic seizures are
*1.Phenobarbital
2.Ethosuximide
3.Carbamazepine
4.Selegiline
5.Cyclodolum
9.Benzodiazepine derivatives used for termination
of status epilepticus is
1.Phenytoin
*2.Diazepam
3.Phenobarbital
4.Valproic acid
5.Nitrazepam
10.Indicate the drug which activates dopaminergic sistem
and inhibits MAO B
*1. Selegilinum
2.Carbidopa
3.Levodopum
4.Nacom
5.Midantanum
11.Indicate the trade name of preparation which is
combination of carbidopa and levodopa
1. Selegilinum
2.Tolcaponum
3.Levodopum
*4.Nacom
5.Midantanum
12.Deficiency in dopamine production by neurons of
striopallidal system is considered to be one of the main causes
of Parkinson s disease. Point out the drug used
for replacement therapy of this disease
1.Atropine
2.Dopamin
*3.Levodopa
4.Carbidopa
5.Diazepam
13.Indicate the drugs used for the preventtion of absence seizures
1.Dipeninum
*2.Ethosuximide
3.Carbamazepine
4.Trimetinum
5.Phenobarbital
14.What drug has antiparkinsonic activity owinq to
central cholinoblockinq action?
1.Levodopa
2.Midantanum
*3.Cyclodolum
4.Ethosuximide
5.Dipeninum
15.Indicate the drugs used for treatment of epileptic status
1.Ethosuximide
2.Carbamazepine
*3.Diazepam
4.Thiopental sodium
5.Clonazepam
@ RESPIRATORY
1.Indicate the antitussive drug with central type of action
1.Libexinum
2.Salbutamolum
*3.Codeinum
4.Bromhexinum
5.Mucaltinum
2.Indicate the expectorant drug
1.Libexinum
2.Salbutamolum
3.Codeinum
4.Ipratropium
*5.Mucaltinum
3.A 13-year-old girl with history of asthma complained
of cough, dyspnea and wheezing. Her symptoms became
so severe that her parents brought her to the emergency room.
Physical examination revealed diaphoresis, dyspnea, tachycardia,
and tachypnea. Her respiratory rate was 42/min, pulse rate
was 110 beats per minute, and blood pressure was 130/70 mm Hg.
Choose the most appropriate drug to rapidly reverse bronchoconstriction:
*1.Salbutamol
2.Cromolyn
3.Beclomethasone
4.Methylprednidsolone
5.Ipratropium
4.A patient with bronchial asthma had been taking tablets
which caused insomnia, headache, increased blood pressure.
What medicine can cause such complications?
1.Chromolin sodium
2.Izadrine
3.Euphyline
4.Adrenaline
*5.Ephedrine
5.A patient suffers from nocturnal paroxysms of bronchial
asthma accompanied by bradycardia, spastic intestinal pains
and diarrhea. Medicine of what group can releave these symptoms?
1.Sympatholytics
2.N-cholinergic receptors
*3.M-cholinergic receptors
4.Saluretics
5.beta-adrenoblockers
6.A patient with bronchial asthma is administered inhalation of 0,5%
isadrine solution. Bronchospasm was releaved, but the patient
began complaining of pain in the heart region and palpitation.
It is connected with the stimulation of:
1.alfa-1 -adrenoreceptors
*2.beta-1 -adrenoreceptors
3.beta-2 -adrenoreceptors
4.Acetylcholine synthesis
5.M-cholinergic receptors
7.Codeine has the following characteristics except:
*1.It is weak antitussive agent
2.It causes drug dependence
3.It inhibits respiratory center
4.It causes constipation
5.It is centrally acting agent
8.The following statements pertaining to glaucine are correct
1.It suppresses respiration
2.It causes development of drug depence
3.It cauces constipation
*4.It decreases BP
5.It increases BP
9.Histamine induced bronchospasm can be relieved by
1.Ranitidine
2.Dobutamine
*3.Adrenaline
4.Libexin
5.Mucaltinum
10.An acute attack of bronchial asthma is treated by
1.Sodium cromoglycate
*2.Salbutamol inhalation
3.Aspirin
4.Codeine
5.Acetylcysteinum
11.The following drug is contraindicated in patients with
bronchial asthma
*1.Propranolol
2.Ipratropium bromide
3.Terbutaline
4.Nifedipine
5.Adrenaline
12.Euphylline acts by
1.inhibition of COX enzyme
*2.inhibition of phosphodiesterase
3.stimulation of adenilate cyclase synthesis
4.stimulation of adenosine reuptake
5.inhibition of adenosine deaminase
13.The following statements pertaining to cromolyn sodium are
correct except
*1.It is used in bronchial asthma due to its bronchodilating effect
2.It alters the function of channels in the cell membrane
3.It inhibits mast cell activation
4.It is used for prophylaxis of bronchial asthma
5.It is antiallergic drug
14.The group of drugs most widely used as bronchodilators is
1.alpha-adrenomimetics
*2.beta-2-adrenomimetics
3.M-cholinoblockers
4.ganglion blocking drugs
5.sympatholytics
15.Glucocorticoids are beneficial in bronchial asthma due to except
*1.direct relaxation of bronchial muscle
2.prevention of leucotriens synthesis
3.decrease of mucosal oedema
4.stabilization of mast cell membrane
5.exert antiallergic effect
@ Antiarrhitmics
1. In a patient who has had attacks of paroxysmal atrial tachycardia, an
ideal prophylactic drug is
1. Adenosine
2. Procainamide
3. Lidocaine
4. Nifedipine
*5. Verapamil
2. Which of the following is an antiarrhythmic agent that has relatively
few electrophysiologic effects on normal myocardial tissue but suppresses
the arrhythmogenic tendencies of ischemic myocardial tissues?
1. Propranolol
2. Procainamide
3. Quinidine
*4. Lidocaine
5. Disopyramide
3. The first-line drug for treating an acute attack of
reentrant supraventricular tachycardia (SVT) is
*1. Adenosine
2. Digoxin
3. Propranolol
4. Phenylephrine
5. Edrophonium
4. A 76-year-old female with an eight-year history of CHF that has been
well controlled with digoxin and furosemide develops recurrence of dyspnea
on exertion. On physical examination, she has sinus tachycardia,
rales at the base of both lungs. Which agent
could be added to her therapeutic regimen?
1. Dobutamine
2. Hydralazine
3. Minoxidil
4. Prazosin
*5. Enalapril
5. A 61-year-old female has intermittent bouts of chest pain on exertion
of two months duration, associated with numbness and tingling in the
fourth and fifth fingers of her left hand.
An EKG is normal. She is placed on propranolol,
which relieves her symptoms.
What cardiovascular effect does
the drug have?
1. It decreased production of catecholamines
2. It dilated the coronary vasculature
*3. It decreased the requirement for myocardial oxygen
4. It increased peripheral vascular resistance
5. It increased sensitivity to catecholamines
6. A 66-year-old man had a myocardial infarction.
Which one of the following would be appropriate
prophylactic antiarrhythmic therapy?
1. Lidocaine
*2. Metoprolol
3. Procainamide
4. Quinidine
5. Verapamil
7. Suppression of arrhythmias resulting from a reentry
focus is most likely to occur if the drug:
1. has vagomimetic effects on the AV node.
2. is a beta-blocker.
*3. converts a unidirectional block to a bidirectional block.
4. slows conduction through the atria.
5. has atropine-like effects on the AV node.
8. A 57-year-old man is being treated for an atrial arrhythmia.
He complains of headache, dizziness, and tinnitus.
Which one of the following antiarrhythmic drugs
is the most likely cause?
1. Amiodarone
2. Procainamide
3. Propranolol
*4. Quinidine
5. Verapamil
9. A 58-year-old woman is being treated
for chronic suppression of a ventricular arrhythmia.
After two months of therapy, she complains about
feeling tired all the time. Examination reveals a
resting heart rate of ten beats per minute lower than her previous rate.
Her skin is cool and clammy. Laboratory test results indicate low thyroxin
and elevated thyroid-stimulating hormone levels.
Which of the following antiarrhythmic drugs is the
likely cause of these signs and symptoms?
*1. Amiodarone
2. Procainamide
3. Propranolol
4. Quinidine
5. Verapamil
10. Which drug from membrane stabilizing agents
of 2nd group also used as local anesthetic?
1. Difenin
2. Mexiletin
3. Disopiramide
4. Novocain
*5. Lidocain
11. All statements about quinidine are correct EXEPT:
1. increases ERP
2. decreases heart contraction
3. decreases automaticity
4. slow down impulse conducting
*5. increases heart contraction
12. Which pharmacological substance is chemically close to novocainum,
used orally or intravenously in different disturbances of heart rate?
1. Difenin
2. Mexiletin
3. Amiodaron
*4. Novocainamide
5. Dultiazem
13. A 52-year-old patient with supraventricular tachyarrhytmia
was using antiarrhitmic drug. For a long period it maintained
the normal cardiac rhythm, but with time dispnoe started to occur
while going upstairs. Examination revealed pulmonary sclerosis.
Which antiarrhythmic drug can cause such side effect?
1. Verapamil
2. Procainamide
*3. Amiodaron
4. Novocainamide
5. Dultiazem
14. To maintaine sinus rhythm in a patient with tachyarrhytmia
dortor prescribed antiarrhitmic drug.
After laboratory testing of blood sugar level
discontinuation of this drug was recomended.
What drug was initially prescribed?
1. Verapamil
*2. Anaprilin
3. Amiodaron
4. Novocainamide
5. Dultiazem
15. Patient with initial form of hypertension complains
on pain in heart and tachycardia.
Which of the following drugs has to be used for
treatment of this patient?
1. Phentolamine
*2. Anaprilin
3. Dihydroergotamine
4. Prazosin
5. Octadinum
@antiHYPERTENSIV drugS
1.Which of the following antiadrenergic drugs used in the treatment
of hypertension is contraindicated in patients prone to bronchospasm?
1.Prazosin
2.Metoprolol
3.Reserpine
4.Atenolol
*5.Propranolol (Anaprilinum)
2.A hypertensive patient was treated with the drug suppress¬ing
the formation of angiotensin II and preventing degradation
of bradykinine. Point out the drug realizing
antihypertensive effect by these mechanisms.
1.Nifedipine
2.Guanethidine
3.Clonidine
*4.Enalapril
5.Propranolol
3.For treatment of hypertensive patient doctor prescribed captopril,
but soon patient addressed doctor, complaining of dry cough and rashes.
Doctor substituted captopril for losartan.
What is the mechanism of the last drug action?
1.Inhibition of renin release
2.Inhibition of ACE
3.Inhibition of angiotensinogen conversion to angiotensin I
*4.Blockade of angiotensin II receptors
5.Increase in bradykinin level
4.A 45-year-old man has recently been diagnosed with hypertension
and started on monotherapy designed to reduce peripheral resistance
and prevent NaCI and water retention. He has developed a persistent cough.
Which of the following drugs would have the same
benefits stimilar MAO but would not cause cough?
*1.Losartan
2.Nifedipine
3.Prazosin
4.Propranolol
5.Clonidine
5.Which one of the following drugs may cause a precipitous
fall in blood pressure and fainting on initial administration?
1.Atenolol
2.Hydrochlorothiazide
3.Nifedipine
*4.Prazosin
5.Verapamil
6. Which one of the following antihypertensive drugs
can precipitate a hypertensive crisis following
abrupt cessation of therapy?
*1.Clonidine
2.Diltiazem
3.Enalapril
4.Losartan
5.Hydrochlorothiazide
7.Doctor prescribed selective alpha1-adrenoceptor blocker
for treatment of hypertensive patient. Which of the
following drugs belongs to this group?
1.Metoprolol
2.Phentolamine
3.Amiodarone
4.Tropaphenum
*5.Prazosin
8. A 54-year-old man suffering hypertension complains
to severe tachycardia after administration of antihypertensive drug.
Which of the following antihypertensive drugs
is most likely to cause reflex tachycardia?
*1.Hydralazine
2.Verapamile
3.Hydrochlorothiazide
4.Propranolol
5.Captopril
9. A 45 year-old hypertensive male has been manifested hypoglycemia
after administration antihypertensive drug.
From the list of antihypertensive drugs below
select the one most likely to lower blood sugar.
*1.Propranolol
2.Prazosin
3.Nifedipine
4.Captopril
5.Hydralazine
10. A 66-year-old mail suffering from arterial hypertension
has accompanying chronic bronchitis with asthmatic component.
Which drug is contraindicated in this situation?
1.Captoprile
*2.Propranolol
3.Nifedipine
4.Hydrochlorothiazide
5.Verapamil
11. A patient suffering hypertension has been treated
with diuretic drug. In a few days blood pressure decreased
but symptoms of hypokalemia has appeared. Which agent
could induce such complication?
*1.Furosemide
2.Triamteren
3.Spironolactone
4.Enalapril
5.Clofeline
12. Angiotensin converting enzyme (ACE) inhibitors are
associated with a high incidence of which of the
following adverse reactions?
1.Hepatitis
2.Hirsutism
3.Agranulocytosis
4.Anemia
*5.HypERkalemia
13. A 58-year-old female with a four-year history of hypertension
controlled on hydrochlorothiazide is found on a physical examination
to have a blood pressure of 155/105 mmHg. The hydrochlorothiazide
is substituted with another agent. 15 days later, she returns for
follow-up complaining of a loss of taste. Which agent could
induce such complication?
1.Propranolol
*2.Captoprile
3.Nifedipine
4.Hydrochlorothiazide
5.Verapamil
14. A 66-year-old female is treated for hypertension with antihypertensive
that controls her blood pressure. One day,
she comes to the ED with chest pain,
tachycardia, anxiety, and a BP of 240/140 mmHg.
She has not taken her medication for 2 days.
Which antihypertensive can account for her findings?
1.Clonidine
*2.Propranolol
3. Captoprile
4.Hydrochlorothiazide
5.Verapamil
15. 43-year-old, insulin-dependent diabetic patient
is diagnosed with hypertension and begins therapy with
an antihypertensive agent. Three days later, he measures
his blood glucose at home and finds that it is 53 mg/dL.
He recalibrates his glucose testing apparatus
and repeats the test, only to find that the first reading was accurate.
He is concerned that his hypoglycemia did not produce
the normal premonitory signs and symptoms.
Which of the following medications was most likely
prescribed to treat his hypertension?
1.Captopril
2.Diltiazem
3.Methyldopa
4.Prazosin
*5.Propranolol
16.Mild hypertention be controlled with a single drug
of listed group, excluding
1.Diuretics
2.b adrenoblockers
3.ACE-inhibitors
*4.K+ channel openers
5.calcium channel blockers
17. The calcium channel blocker with higher affinity
to myocardial calcium channels than to those of vessels is
*1.Verapamil
2.Nifedipine
3.Amlodipine
4.Nicorandil
5.Minoxidil
18. The most widely used diuretic agent in the treatment
of hypertension belongs to the group of
*1.Benzothiazides
2.Loop diuretics
3.Osmotic diuretics
4. Carboanhydrase inhibitors
5. Potassium sparing diuretics
@ Blood
1. A patient complains of anorexia, decrease of weight,
pain in epigastric region. On the bases of clinical data
and blood analysis the disease of Addison-Birmer
(pernicious megaloblastic anaemia) was diagnosed.
What drug should be used in this case?
1.Vicasolum
*2.Cyanocobalaminum
3.Fercovenum
4.Fraxiparin
5.Ferri sulfas
2. Determine the enzyme drug, which causes
degradation of blood clots.
1.Lydazum
2.Pancreatinum
*3.Fibrinolysin
4.Trypsinum
5.Cholinesterase
3. A patient for a long time applied Laevomycetinum.
During laboratory investigation
leucopenia was diagnosed. What drug should be used
for correction of leukopoesis?
1.Metotrexate
*2.Methyluracilum
3.Mercaptopurinum
4.Cyanocobalaminum
5.Prednisolonum
4. On discontinuation of heparin treatment of patient
with myocardial infarction administration of
warfarin was started. This drug exerts its anticoagulant
effect by
1.Blocking calcium binding to clotting factors
2.Forming a complex with clotting factors
3.Breaking down thrombin
*4.Inhibiting of pro-clotting factor synthesis in the liver
5.Depolymerization of fibrin
5. The antithrombotic drug which inhibits cyclooxygenase enzymes is:
1.Prednisone
2.Dipyridamole
3.Tranexamic acid
*4.Aspirin
5.Ticlopidine
6. A 22-year-old woman who experienced pain and swelling
in her right leg presented at the emergency room.
An ultrasound study showed thrombosis in the popliteal vein.
The patient, who was in her second trimester of pregnancy,
was treated for seven days with intravenous unfractionated heparin.
The pain resolved during the course of therapy, and the patient
was discharged on day eight. Which one of the following drugs
would be most appropriate out-patient follow-up therapy for this patient,
who lives 100 miles from the nearest hospital?
1.Warfarin
2.Aspirin
3.Alteplase
4.Unfractionated heparin
*5.Low-molecular-weight heparin (LMWH)
7.A 60-year-old man is diagnosed with deep-vein thrombosis.
The patient was treated with a bolus of heparin, a
nd a heparin drip was started. One hourlater,
he was bleeding profusely from the intravenous site.
The heparin therapy was suspended, but the bleeding continued.
Protamine was administered intravenously
and the bleeding resolved. The protamine:
1.degraded the heparin.
2.inactivates antithrombin.
3.activates the coagulation cascade.
4.activates tissue-plasminogen activator.
*5.ionically combines with heparin.
8.A 54-year-old male with a prosthetic aortic valve
replacement complained to his family physician
of black and tarry stools. Physical examination and
vital signs were unremarkable except for
subconjunctivial hemorrhages and bleeding gums.
Stools tested positive for heme and hematuria was observed.
The patient has been receiving oral warfarin since his valve
replacement one year earlier. Prothrombin time was found
to be significantly elevated. Which one of the following
therapies would provide the most rapid recovery
from the observed bleeding secondary to warfarin treatment?
1.Intravenous vitamin K
*2.Transfusion of fresh-frozen plasma
3.Intravenous protamine
4.Immediate withdrawal of warfarin treatment
5.Intravenous administration of anti-warfarin anti-bodies
9.A 60-year-old male, following hospitalization for
an acute myocardial infarction, is treated with warfarin.
What is the mechanism of its action?
1.Increase in the plasma level of factor IX
2.Inhibition of thrombin and early coagulation steps
*3.Inhibition of synthesis of prothrombin and
coagulation factors VII, IX, X
4.Inhibition of platelet aggregation in vitro
5.Activation of plasminogen
10.A 39-year-old pregnant female requires heparin
for thromboembolic phenomena.
What is the mechanism of its action?
1.Increase in the plasma level of factor IX
*2.Inhibition of thrombin and early coagulation steps
3.Inhibition of synthesis of prothrombin and
coagulation factors VII, IX, X
4.Inhibition of platelet aggregation in vitro
5.Activation of plasminogen
11.A 47-year-old female comes to the emergency department (ED)
with severe crushing chest pain of one hour s
duration. Electrocardiogram and blood tests proved acute MI.
Streptokinase is chosen as part of the
therapeutic regimen. What is the mechanism of its action?
1.It activates the conversion of fibrin
to fibrin-split products
*2.It activates the conversion of plasminogen to plasmin
3.It inhibits the conversion of prothrombin to thrombin
4.It inhibits the conversion of fibrinogen to fibrin
12.A 42-year-old male with an acute MI is treated with alteplase.
What is the mechanism of action of alteplase?
1.Inhibition of platelet thromboxane production
2.Antagonism of ADP receptor
3.Glycoprotein IIb/IIIa antagonist
4.Inhibition of the synthesis of vitamin K dependent
coagulation factors
*5.Activation of plasminogen from plasmin
13.Which of the following anemias would be treated with cyanocobalamin
(vitamin B12)?
1.Anemia in infants who are undergoing rapid growth
*2.Anemia associated with cheilosis, dysphagia,
gastritis, and hypochlorhydria
3.Anemia associated with small, bizarre cells
poorly filled with hemoglobin
4.Anemia associated with infestation by Diphyllobothrium latum
5.Bleeding from a gastric ulcer
14.The direct acting plasminogen activator is:
*1.Streptokinase
2.Vitamin K
3.Epsilon-aminocapronic acid
4.Antistreplase
5.Heparin
15.Heparin is:
1.Coagulant
*2.Anticoagulant
3.Antifibrinolytic
4.Fibrinolytic
5.Antihypertensive
16.Vitamin K is used in:
1.Diseases of GIT
2.Diseases of kidneys
3.Before operations
4.Thromboembolia
*5.Capillary and parenhimatic bleedings
17.Side effects of cyancobalamine:
*1.Allergic reactions
2.Constipation
3.Peptic ulceration
4.Staining of teeth
18.Drug for treatment of acute iron poisoning:
1.Pralidoxime
*2.Desferrioxamine
3.Atropine
4.Flumazenil
5.Ferrous succinate
19.What factor can increase absorbtion of iron for GIT:
*1.Ascorbic acid
2.Milk
3.Phosphates
4.Tetracyclines
5.Alkalies
20.The primary use of erythropoietin:
*1.Anaemia of chronic renal failure
2.Megaloblastic anaemia
3.Hypochromatic anaemia
4.Hyperchromatic anaemia
5.Cronic alcoholism
@ ANTIANGINAL
1.Which of the following is involved in antianginal
action of nitroglycerin?
1.Beta-adrenergic activity
2.Phosphodiesterase activity
3.Alpha-adrenergic activity
*4.cGMP
5.cAMP
2.Which of the following hemodynamic effects of nitroglycerin
are primarily responsible for the beneficial results, observed
in patients with secondary angina?
*1.Reduction in venous capacitance and systemic vascular resistance
2.Reduction in the force of myocardial contraction
3.Have an antiarrhythmic effect
4.Increased heart rate
5.Increased blood flow to the subepicardium
3.A patient after tooth extraction developed persistent substernal
pain. Sublingual antianginal substance releaved the substernal pain,
but the patient complained of headache and dizziness. What medicine
did the patient use?
1.Validol
*2.Nytroglycerin
3.Metoprolol
4.Anapriline
5.Verapamil
4.Nitroglycerine is used sublingually because
1.It is not absorbed from GIT
2.It is destroyed by gastric juice
3.It interacts with components of food
*4.It undergoes intensive hepatic first pass metabolism
5.All listed
5.A 69-year-old-woman was prescribed a combination of drugs consisting of
a nitroglycerin patch and a beta-adrenoblocker, such as propranolol, to
treat her attacks of secondary angina. Which effect of propranolol would
counteract an adverse effect of nitroglycerin?
*1.A decrease in heart rate
2.A decrease in preload
3.A decrease in afterload
4.An increase in myocardial contractile force
5.A reduction in coronary vasospasm
6.A 62-year-old man with angina pectoris develops severe constipation
following treatment with:
*1.Verapamil
2.Nitroglycerin
3.Metoprolol
4.Anapriline
5.Dobutamine
7.Nitroglycerine causes all listed effects except:
*1.delay A-V condaction
2.reflex tachycardia
3.tolerance
4.hypotension
5.headache
8.The beneficial effects of nitrates in angina pectoris are except:
1.preload decrease
2.afterload decrease
3.relaxation of bigger coronary arteries
4.increase in oxygen supply
*5.increase in systemic BP
9.The following long nitrates exept:
1.Isosorbite mononitrate
2.Isosobite dinitrate
3.Erythrityl tetranitrate
4.Pentaerythritol tetranitrate
*5.Nitroglycerine
10.Clinical uses of nitrates include all except
1.angina pectoris
2.congestive heart failure
3.myocardial infaction
4.percutaneous coronary angioplasty
*5.renal insufficiency
11.The following statements pertaining to dipyridamole are
correct except
1.It is powerful coronary vasodilator
2.It may worsen blood supply of ichemic area
due to "coronary steal phenomenon"
3.It is stimulates prostacycline synthesis by endothelium
4.It exerts antiplatelet effect
*5.It is block COX enzyme
12.A 56-year-old patient complains of chest pain following any
sustained exercise.He is diagnosed with atherosclerotic angina.
He is prescribed sublingual nitroglycerin for treatment
of acute chest pain.Which of the following adverse effects is
likely to be experienced by this patient?
1.Hypertension
*2.Throbbing headache
3.Bradycardia
4.Sexual dysfunction
5.Anemia
13.The patient 60-year-old complains of chest pain following any
sustained exercise.He is diagnosed with atherosclerotic angina.
He is prescribed sublingual nitroglycerin for treatment and also
propranolol to prevent episodes of angina. The beta-blocker
has the added benefit of preventing which of the following side
effects of sublingual nitroglycerin?
1.Dizziness
2.Methemoglobinemia
3.Throbbing headache
*4.Reflex tachycardia
5.Edema
14.A 68-year-old man has been successfully treated for
exercise-induced angina for several years. He recently has
been complaing about being awakened at night with chest pain.
Which of the following drugs would be useful in preventing
this patient's nocturnal angina?
1.Riboxinum
2.Pentoxyphylline
*3.Isosorbidi mononitras
4.Dipyridamole
5.Validolum
@ DIURETICS
1.A patient suffering from hypertension has been treated
with diuretic drug. In a few days blood pressure decreased
but symptoms of hypokalemia has appeared. Which agent could
induce such complication?
*1.Furosemide
2.Triamterene
3.Spironolactone
4.Amiloride
5.Enalaprilum
2.A 50-year-male with pitting edema of the ankles
develops gynecomastia and erectile dysfunction while
being treated with which of the following agents?
1.Hydrochlorothiazide
2.Furosemide
*3.Spironolactone
4.Triamterene
5.Amiloride
3.Of the following agents, which is best avoided in
a patient with a history of chronic congestive heart failure?
1.Spironolactone
2.Triamterene
*3.Mannitol
4.Hydrochlorothiazide
5.Furosemide
4.Furosemide inhibits the sodium-potassium-dichloride
co-transporters that are located in the:
1.Collecting duct
*2.Ascending limb of the loop of Henle
3.Descending limb of the loop of Henle
4.Proximal tubule
5.Distal convoluted tubule
5.A reduction in insulin release from the pancreas may
be caused by which of the following agents?
1.Amiloride
2.Hydrochlorothiazide
3.Mannitol
4.Spironolactone
*5.Furosemide
6.A 60-year-male with a history of chronic congestive
heart failure develops hearing loss while being treated
with which of the following agents?
*1.Ethacrynic acid
2.Mannitol
3.Chlorothalidone
4.Spironolactone
5.Amiloride
7.A 65-year-old female has hypertension, for which she is
receiving hydrochlorothiazide to lower her blood pressure.
Which of these ions would not increase in concentration in
her urine?
1.K+
2.Cl-
*3.Ca2+
4.Na+
5.Mg2+
8.A doctor administered Allopurinol to a 26-year-old young
man presenting with the symptoms of gout. What pharmacological
action of Allopurinol ensures therapeutically effect?
*1.inhibition of uric acid synthesis
2.increase excretion solts of uric acid
3.inhibition of leucocyte migration into the joint
4.general anti-inflammatory effect
5.general analgesic effect
9.Indicate the mechanism of urodanum as antigout action:
1.Inhibition of synthesis of uric acid
2.Stimulation of of uric acid destruction
3.Increase of of uric acid elimination by kidneys
*4.Formation of complexes with uric acid
5.Inhibition of enzyme xantineoxydase
10. Indicate the drug for weakening the contractile activity
of the pregnant uterus which is used in danger of premature
labour:
1.Oxytocin
*2.Fenoterol
3.Ergometrine
4.Pituitrin
5.Dinoprost
11.An elderly patient with a history of heart disease and who
is having difficulty breathing is brought into the emergency room.
Examination reveals that she has pulmonary edema.Which of the
following treatments is indicated?
1.Spironolactone
*2.Furosemide
3.Hydrochlorothiazide
4.Triamterene
5.Amiloride
12.An alcoholic male has developed hepatic cirrhosis. In order
to control the ascites and edema, he is prescribed which one
of the following:
1.Hydrochlorothiazide
*2.Spironolactone
3.Furosemide
4.Mannitum
5.Urodanum
13.A 55-year-old male with kidney stones has been placed on
a diuretic to decrease calcium exretion. However, after a few
weeks, he develops an attack of gout. Which diuretic was he taking?
1.Triamterene
2.Spironolactone
3.Furosemide
*4.Dichlothiazidum
5.Allopurinolum
14.An 55-year-old woman with hypertension is being
treated with a thiazide. Her blood pressure responds and reads
at 120/76 mm Hg. After several months on the medication,
she complains of being tired and weak.
An analysis of the blood indicates low values
for which of the following?
1.Calcium
2.Uric acid
*3.Potassium
4.Sodium
5.Glucose
15.Therapeutic uses of furosemide include the following except
1.acute pulmonary oedema
2.renal failure
*3.hypokalemia
4.hypertension
5.refractory oedema
16.Thiazides produce the following effects except
1.hypokalemia
2.hypercalcemia
3.hyperglycemia
4.hyperuricemia
*5.hypernatremia
17.The followin statements concerning osmotic diuretics
are correct except
*1.Osmotic diuretics act mainly in the collecting tubule
2.Mannitol is inert chemically
3.They reduce intracranial pressure
4.They are not reabsorbed from the nephrone
5.They reduce intraocular pressure
@ENDOCRINE DRUGS.
1. Glibenclamide was indicated to a patient with diabetes.
Determine the mechanism of action of the drug.
1. Intensifies glycogenolysis
2.Decreases utilization of glucose by cells
*3.Activates alfa-cells of pancreas
4.Stimulates gluconeogenesis
5.Stimulates hypotalamic centres
2. A man suffers from chronic adrenal insufficiency
(Addison s disease). Indicate a drug for him.
1. Corticitropinum
2. Noradrenalin hydrotartras
3. Prednisolonum
4. Adrenalinum hydrochloridum
*5. Desoxycorticosteronum acetas
3.Insulin was indicated to a patient with diabetes mellitus.
Determine the mechanism of action of the drug.
1.Intensifies glycogenolysis
2.Activates alfa-cells of pancreas
3.Stimulates gluconeogenesis
4.Stimulates hypotalamic centres
*5.Improves utilisation of glucose by cells
4.A patient with diabetes mellitus lost his consciousness.
Objectively: paleness of skin, weak pulse,
AP 100/60, odour of acetone in breath (signs of hyperglycaemic coma).
What drug should be injected
to the patient?
1. Glibenclamide
2. Prednisolonum
3. Metforminum
4. Adrenalini hydrochloridum
*5. Insulin
5. Determine the posterior pituitary hormone which
is used for stimulation of weak labour.
1.Ergometrini maleate
2.Dinoprost
3.Fenoterol
4.Salbutamolum
*5.Oxytocinum
6.Hyperthyroidism can be treated by all but which one
is the best of the following?
*1.Triiodothyronine
2.Surgical removal of the thyroid gland
3.Iodide
4.Propylthiouracil
5.Methimazole
7.Osteoporosis is a major adverse effect caused by the glucocorticoids.
It is due to their ability to:
1.increase the excretion of calcium.
*2.inhibit absorption of calcium.
3.stimulate the hypothalamic-pituitary-adrenal axis.
4.decrease production of prostaglandins.
8. A child with asthma is being treated effectively
with an inhaled preparation of
beclomethasone dipropi-onate. Which of the following
adverse effects is of particular concern?
1.Hypoglycemia
2.Hirsutism
*3.Growth suppression
4.Cushing syndrome
5.Cataract formation
9. A patient with infectious mononucleosis had been
taking glucocorticoids for two weeks.
He was brought into remission, but he fell ill
with acute attack of chronic tonsillitis.
What action of glucocorticoids caused this complication?
1. Antiallergic
*2.Immunosuppressive
3.Antitoxic
4.Antishock
5.Anti-inflammatory
10. Taking orally contraceptives containing sex hormones
inhibits endogenous pituitary secretion.
From the list of hormones below select
the one which secretion is inhibited at taking orally
contraceptives containing sex hormones.
*1.Follicle-stimulating hormone
2.Oxytocin
3.Pituitrin
4.Estrogen
5.Progestin
11. A 7-year-old boy was complaining of thirst and
frequent urination. On examination hyperglycemia
was revealed and diabetes mellitus was diagnosed.
Which of antidiabetic drugs most probably can be
used for treatment of this patient?
1. Acarbose
*2.Insulin
3.Glibenclamide
4.Metformin
5.Clorpropamide
12. Indicate preparation of the posterior pituitary hormones:
1.Corticotropin
*2.Oxytocin
3.Insulin
4.Growth hormone
5.Thyroid-stimulating hormone
13. A patient has been taking glucocorticoids for a
long time. The symptoms of disease
aggravation, decreased blood pressure and weakness were
appeared after preparation withdrawal.
What is the reason of this condition?
*1.Appearance of adrenal insufficiency
2.Hyperproduction of ACTH
3.Sensibilization
4.Habituation
5.Cumulation
14. The patients was admitted to the reception ward
with complains of increased heart beat,
weakness, shivering of muscles, increased sweating,
patient suffers from diabetes mellitus
since 10 years old, takes hypoglycaemic drugs regularly.
Indicate drug of urgent aid:
1.Glucose solution
*2.Insulin
3.Sodium Chloride
4.Magnesium Sulphas
5.Adrenalin
15. Indicate preparation of the anterior pituitary hormones:
*1.Corticotropin
2.Oxytocin
3.Adiurecrin
4.Pituitrin
5.Vasopressin
16.Indicate the antythyroid drug:
1.Thyrotropin
2.Thyroiidin
*3.Mercasolilum
4.Triiodthyronin
5.L-thyroxin
@ VITAMINS. ENZYMATIC AND ANTIENZYMATIC DRUGS.
1. In 3 years old girl the disturbances of bone ossification,
deformations of spinal column, thorax and legs are revealed.
Which drug is most effective in this case?
1. Adiurecrinum
2. Tocopheroli acetas
*3. Ergocalciferolum
4. Retinoli acetas
5. Thiamini chloridum
2. In a man after operation of stomach extraction the level of
erythrocytes is 2,0x1012 per litre, Hb 85 g/l, colour index 1,27.
The deficiency of which
vitamin should be corrected in this case?
1. Vitamin Ð
2. Vitamin À
3. Vitamin Â6
4. Vitamin Ñ
*5. Vitamin Â12
3. A patient is suffering from pain along nerves
and increased content of piruvate
in blood. The deficiency of which vitamin
should be corrected in this case?
1. Vitamin Ñ
2. Vitamin D
3. Vitamin A
*4. Vitamin Â1
5. Vitamin E
4. A patient has passed a course of radiation therapy.
Which vitamin drug with antioxidant properties should
be indicated to the patient for normalization
of vessel permeability?
1. Thiamini chloridum
2. Pyridoxini hydrochloridum
*3. Ascorutinum
4. Cyanocobalaminum
5. Acidum folicum
5. A vitamin drug with significant vessel-dilating
action was used in a complex
treatment of parodontosis. Determine the drug.
1. Acidum ascorbinicum
2. Thiamini chloridum
*3. Acidum nicotinicum
4. Retinoli acetas
5. Pyridoxini hydrochloridum
6. A patient is suffering from dermatitis, diarrhea and dementia.
Pellagra was diagnosed. Which vitamin drug
should be used for treatment?
1. Thiamini chloridum
2. Tocopheroli acetas
*3. Acidum nicotinicum
4. Retinoli acetas
5. Riboflavinum
7. The disbacteriosis of intestine was diagnosed in patient.
The synthesis of which vitamin is oppressed?
1. Vitamin À
*2. Vitamin Â1
3. Vitamin Ñ
4. Vitamin D
5. Vitamin E
8. A patient with rachitis for a long time applied vitamin D,
which caused such side effects as bones demineralisation,
Ca storage in vessels, kidneys, liver, heart,
lungs and intestines (calcinosis).
Which vitamin can eliminate these side effects
of vitamin D?
1. Vitamin Â6
2. Vitamin Â12
3. Vitamin ÐÐ
4. Vitamin Â1
*5. Vitamin À
9. A polyneuritis developed in chronic alcoholic.
What vitamin should be indicated in this case?
1. Rutinum
2. Ergocalciferolum
3. Thiamini chloridum
*4. Acidum ascorbinicum
5. Riboflavinum
10. In a man of 39 years old the hemeralopia,
xerophthalmia and impairment of skin
epithelium are observed. What vitamin should be
indicated in this case?
1. Retinoli acetas
2. Pyridoxini hydrochloridum
*3. Riboflavinum
4. Ergocalciferolum
5.Tocopheroli acetas
11. Fat-soluble vitamins generally have a greater
potential toxicity compared with
water-soluble vitamins because they are
1. More essential to vital metabolic processes
2. Metabolically faster
*3. Avidly stored by the body
4. Administered in larger doses
5. Involved in more essential metabolic pathways
12. Which of the following vitamin in large doses is teratogenic?
*1. Vitamin A
2. Vitamin B12
3. Vitamin C
4. Vitamin B6
5. Vitamin E
13. Vitamin K is used in:
1. Diseases of GIT
2. Diseases of kidneys
3. Diseases of respiratory organs
4. Thromboembolism
*5. Capillary and parenchymatic bleedings
14. Determine which side effect is most common for cyancobalamine:
*1. Allergic reactions
2. Constipation
3. Peptic ulceration
4. Nausea, vomiting
5. Staining of teeth
15. In the treatment of neurological pathology the
following vitamin preparation
is commonly used:
1. Retinol
2. Tocoferol
*3. Thiamine
4. Vikasol
5. Calciferol
16. Loosening of teeth, gingivitis and petechial
hemorrhage are the symptoms of the deficiency
1. Vitamin A
2. Vitamin B12
*3. Vitamin C
4. Vitamin B6
5. Vitamin E
17. Antioxidant vitamins include the following except:
1. Vitamin A
*2. Vitamin B1
3. Vitamin C
4. Vitamin P
5. Vitamin E
18. The following statements concerning lydasum are correct except:
1. It is hialuronidase
2. It is used in treatment of collagen diseases
*3. It is used for replacement therapy
4. It is used to promote drug absorption
5. It is fibrinolytic drugs
@Anti-inflammatory drugs
1. The antithrombotic drug which inhibits cyclooxygenase enzymes is:
1. Prednisone
2. Dipyridamole
3. Dimidrolum
*4. Aspirinum
5. Analginum
2. Indicate a patient the drug, which blocks H1-histamine
receptors and doesn t
obtain sedative action:
*1. Loratadine
2. Dimedrolum
3. Adrenalini hydrochloridum
4. Salbutamolum
5. Prednisolone
3. What drug stabilizes basophils membranes,
prevents release of allergy mediators
and significantly dilates bronchi?
*1. Cromolyn sodium
2. Isadrinum
3. Dimedrolum
4. Salbutamolum
5. Morphini hydrochloridum
4. A doctor prescribed cromolyn sodium for prevention
of bronchial asthma attacks.
What is the mechanism of action of this drug?
1. Decreases the level of immunoglobulins
2. Blocks Í1-histaminic receptors
*3. Stabilizes basophils membranes
4. Binds free histamine
5. Blocks Í2-histaminic receptors
5. A patient is a bus driver. He needs an antialergic drug,
which doesn t influence on CNS.
Determine this drug.
1. Diprazinum
2. Dimedrolum
3. Diazolinum
4. Suprastin
*5. Ketotifenum
6. In a patient with cancer after the radiation therapy
the decrease of indexes of cell immunity was revealed.
Indicate a drug from a group of immunostimulators
of animal origin.
1. Levamisolum
2. Interferon
3. Prodigiosanum
*4. Thymalinum
5. Reaferon
7. A man of 62 years old for a long time suffers from coxitis.
A doctor indicated a new
nonsteriod anti-inflammatory drug Piroxicam .
What is the mechanism of action of this drug?
1. Inhibition of phosphodiesterase
2. Activation of adenilatecyclase
3. Inhibition of cholinesterase
*4. Inhibition of cyclooxygenase
5. Activation of phospholipase
8. A patient with reumatoid arthritis for a
long time applied Indomethacin.
Some time later the signs of gastritis were revealed.
What action of this drug can explain the complication?
*1. Anticyclooxygenase
2. Antiserotonin
3. Antihistamine
4. Antikinine
5. Local irritating
9. A patient is suffering from an allergic rhinitis.
Determine the drug that should be indicated in this case.
1. Ranitidine
2. Ephedrini hydrochloridum
3. Salbutamol
4. Insulin
*5. Loratadine
10. What drug is used in allergic states: dermatitis,
hay fever, bites of bees and obtains
significant sedative and hypnotic effect?
1. Analginum
2. Atropini sulfas
*3. Dimedrolum
4. Paracetamolum
5. Dichlothiazidum
11. A patient with osteoarthrosis and peptic ulcer of
stomach was indicated an anti-inflammatory drug, which
selectively blocks cyclooxygenase-2. Determine this drug.
*1. Celecoxibum
2. Acidum acetylsalicylicum
3. Diclophenac sodium
4. Acidum mephenamicum
5. Ibuprofen
12. For treatment of systemic scleriasis appointed immunodepressant,
known yet as antimalarial agent.What drug appointed to the patient?
*1. Chingaminum
2. Prednisolonum
3. Dexometsonum
4. Azathioprinum
5. Cycloserinum
13. Indicate a patient the drug, which blocks H1-histamine receptors
and doesn t obtain sedative action:
*1. Loratadine
2. Dimedrolum
3. Adrenalini hydrochloridum
4. Salbutamolum
5. Prednisolone
14. An allergic reaction of immediate type developed in a patient.
Indicate the most effective remedy.
*1. Prednisolone
2. Isadrinum
3. Atropini sulfas
4. Ephedrini hydrochloridum
5. Loratadine
15. According to the laboratory test an immunodefficient
state is diagnosed in a patient.
Which drug should be indicated to a
patient for stimulation of immunity?
1. Phenasal
2. Piperasinum
*3. Levamisolum
4. Benzylpenicillinum-natrium
5. Prednisolonum
16. A patient with lingering bronchopneumonia was
hospitalised to the therapeutical department.
Antibiotic therapy was not effective. What drug should be
used for stimulation of immunity?
1. Analginum
2. Sulfocamphocainum
3. Dimedrolum
4. Paracetamol
*5. Thymalinum
17. In which one of the following conditions would
aspirin be contraindicated?
1. Myalgia
2. Fever
*3. Peptic ulcer
4. Rheumatoid arthritis
5. Unstable angina
18. Which one of the following statements
concerning COX-2 inhibitors is correct?
1. The COX-2 inhibitors show greater analgesic activity
than traditional NSAIDs.
2. The COX-2 inhibitors decrease platelet function.
3. The COX-2 inhibitors do not affect the kidney.
*4. The COX-2 inhibitors show anti-inflammatory activity
similar to that of the traditional NSAIDs.
5. The COX-2 inhibitors are cardioprotective.
19. A 70-year-old man has a history of ulcer disease.
He has recently experienced
swelling and pain in the joints of his hands.
His physician wants to begin therapy with
a non-steroidal anti-inflammatory drug.
Which one of the following drugs might also be
prescribed along with the NSAID in order to reduce
the risk of activating this patient's ulcer disease?
1. Allopurinol
2. Colchicine
*3. Misoprostol
4. Probenecid
5. Sulindac
23. Which one of the following statements
concerning H1 -histamines is correct?
*1. Second-generation H1 antihistamines
are relatively free of adverse effects.
2. Because of the established long-term safety
of first-generation H1 antihistamines, they are
the first choice for initial therapy.
3. The motor coordination involved in driving an
automobile is not affected by the use of first-generation
H1 antihistamines.
4. H1 antihistamines can be used in the treatment of
acute anaphylaxis.
5. Both first-generation and second-generation H1
antihistamines readily penetrate the blood-brain barrier.
@Electrolite balance
1. Which drug is used for normalisation of water-salts metabolism,
osmotic pressure, volume of blood,
increases antitoxic properties of organism?
*1.Magnesii sulfas
2.Calcii chloridum
3.Kalii chloridum
4.Natrii hydrocarbonas
5.Natrii chloridum
2. Which drug is used for normalisation of digestion in
antacid gastritis?
1.Unithiolum
2.Calcii chloridum
3.Trypsinum crystallisatum
*4.Natrii hydrocarbonas
5.Piracetam
3. To mark drug which uses for treatment of local inflammatory
and allergic reactions:
1.Magnesii sulfas
*2.Calcii chloridum
3.Kalii chloridum
4.Natrii hydrocarbonas
5.Natrii chloridum
4. Which of the following substances is most likely
to cause systemic alcalosis:
*1.Sodium bicarbonate
2.Sodium phosphate
3.Castor oil
4.Mineral oil
5.Aluminium hydroxide gel
5. Which acid is used orally in cases of achylia, hypoachylia,
for improvement of absorption drugs of iron?
*1.Acidum hydrochloridum
2.Acidum boricum
3.Acidum salicylicum
4.Acidum citricum
5.Acidum etacrynicum
6. Which drug obtains antimicrobial and antifungal
action in local application;
antacid and expectorant action in oral application;
in intravenous injection corrects acid-base balance?
1.Unithiolum
2.Lidocaini hydrochloridum
3.Trypsinum crystallisatum
*4.Natrii hydrocarbonas
5.Piracetam
7. Determine the drug which decreases viscosity of hyaluronic acid
thus increases tissue permeability and is used in
joint contractures and scars?
*1.Lydazum
2.Pancreatinum
3.Fibrinolysin
4.Trypsinum
5.Cholinesterase
8. Determine the enzyme drug, which causes
degradation of blood clots.
*1.Lydazum
2.Pancreatinum
3.Fibrinolysin
4.Trypsinum
5.Cholinesterase
9. A patient with acute bronchitis was indicated
a photeolytic enzyme as an expectorant.
This enzyme splits fibrinogenous formations
and dilutes viscous exudates. Determine the drug.
1.Streptokinase
*2.Trypsinum
3.Amylase
4.Contrycal
5.Lydazum
@ ANTISEPTICS AND DISINFECTANTS
1.Indicate the antiseptics which belongs to halogen
*1.Chlorhexidini bigluconas
2.Aethacridini lactase
3.Ammonium causticum
4.Argenti nitras
5.Furacilinum
2.Indicate the antiseptics which belongs to oxidizer
1.Aethacridini lactase
2.Argenti nitras
3.Ammonium causticum
*4.Kalii permanganas
5.Chloraminum
3.Indicate the antiseptics which belongs to dye
1.Chlorhexidini bigluconas
*2.Viride nitentis
3.Kalii permanganas
4.Argenti nitras
5.Furacilinum
4.Indicate the mechanism of antimicrobial action of Kalii permanganas
1.Blockade of sulfhydryl group of microbial enzymes
2.Denaturation of microbial enzymes
3.Dehydration of microbial cells
*4.Oxidation of microbial cells
5.Detergent action upon microbial cells
5.Indicate the mechanism of antimicrobial action of Ammonium causticum
1.Blockade of sulfhydryl groups of microbial enzymes
*2.Denaturation of microbial enzymes
3.Dehydration of microbial cells
4.Oxidation of microbial cells
5.Detergent action upon microbial cells
6.Indicate the mechanism of antimicrobial action of Aethonium
1.Blockade of sulfhydryl groups of microbial enzymes
2.Denaturation of microbial enzymes
3.Dehydration of microbial cells
4.Oxidation of microbial cells
*5.Detergent action upon microbial cells
7.Indicate the drug used for disinfection of metal surgical instruments
1.Chlorhexidini bigluconas
*2.Spiritus aethylicus
3.Ammonium causticum
4.Argenti nitras
5.Furacilinum
8.Indicate the drug used for disinfection of articles of patient care
1.Chlorhexidini bigluconas
*2.Chloraminum
3.Kalii permanganas
4.Argenti nitras
5.Furacilinum
9.Patient with abscess of the cut wound was
admitted to traumatological department.
In order to clean the pus doctor washed it
with 3% hydrogen peroxide. Foam was absent.
What caused the absence of the drug activity?
1.Pus in the wound
2.Shallow wound
3.Low concentration of H2O2
*4.Inherited insufficiency of catalase
5.Inherited insufficiency of erythrocyte
phosphatedehydrogenase
10.The following statements concerning disinfectants are
correct, except
1.They are used to kill pathogens on the inanimate objects
2.They have wide spectrum of anti microbial activity
3.They exert bactericidal effect
4.They have non selective action
*5.They are used to kill pathogens inside of human body
@ANTIBIOTICS
1.An 11 year-old boy with pneumonia was treated with amoxicillin. What is the me
chanism of action of this drug?
1.Disturbances of function of cell mem¬brane
*2.Inhibition of cell wall synthesis
3.Inhibition of protein synthesis
4.Inhibition of nucleic acid synthesis
5.Inhibition of folic acid metabolism
2.Which of the following antibiotics is most closely associated with the develop
ment of renal and ototoxicity?
*1.kanamycin
2.penicillin G
3.azithromycin
4.Tetracycline
5.doxycycline
3.A patient with meningitis was indicated a monobactam antibiotic. Determine the
drug.
1.Ceftriaxone
2.Imipenem
3.Oxacillin
*4.Aztreonam
5.Tetracycline
4. A patient with meningitis was indicated a monobactam antibiotic. Determine th
e drug.
1.Ceftriaxone
2.Imipenem
3.Oxacillin
*4.Aztreonam
5.Tetracycline
5.A 25-year-old male returns home from a holiday in the Far East and complains o
f
three days of dysuria and a purulent urethral discharge. You diagnose this to be
a case of gonorrhea.
Which of the following is appropriate treatment?
*1.Ceftriaxone
2.Imipenem
3.Oxacillin
4.Aztreonam
5.Tetracycline
6.Which one of the following is the best route of adminis¬tration/dosing schedule
for treatment
with aminoglyco¬sides based on the drug's concentration-dependent killing property
?
1.Oral every 8 hours
2.Oral every 24 hours
3.Parenterally by continuous intravenous infusion
4.Parenterally every 8 hours
*5.Parenterally every 24 hours
7.A 22-year-old woman had her first prenatal visit. Her physical examination was
normal for a woman
at 12 weeks' gestation. Both the nontreponemal and fluorescent treponemal antib
ody tests were positive.
She denied previous treatment for syphilis. She could not recall signs or sympto
ms of primary or secondary
syphilis in the past year. She had no previous syphilis serology tests for purpo
ses of comparison.
Which of the following would be the best treatment for the patient?
1.Doxycycline
*2.Benzylpenicillin
3.Rifampicin
4.Streptomycin
5.Tetracycline
8.A 60-year-old patient was admitted to the surgical department because of infec
tion caused by blue pus
bacillus (Pseudomonas aeruginosa) which is sensative to penicillin antibiotics.
Indicate which of the given
penicillins has marked activity to the Pseudomonas aeruginosa?
1.Methicillin
2.Oxacillin
3.Benzylpenicillin
4.Phenoxymethylpenicillin
*5.Carbenicillin disodium
9. What are the primary antibiotics used in the treatment of tuberculosis?
1.semisynthetic penicillins
2.cephalosporins
3.tetracyclines
4.chloramphenicols
*5.rifampicins
10. Gram-negative bacteria are typically less susceptible to the effects of natu
ral penicillin than
are Gram-positive bacteria because
1. they lack a transport system for penicillin
2.they degrade penicillin by means of penicillinase enzymes
*3.they lack peptidoglycan in their cell wall
4.their peptidoglycan is chemically modified to be resistant to attack by p
enicillin
5.their outer membrane is an effective permeability barrier to the penicill
in molecule.
11. Identify the common mechanism of action of the aminoglycoside, macrolide and
tetracycline
antibiotics in bacterial cells.
1. growth factor analogs that interfere with procaryotic vitamin metabolism
2.disruption of the outer membrane of Gram-negative bacteria
3.inhibition of cell wall synthesis
*4.interference with translation
12. The selective toxicity of rifamycin is achieved on this basis.
1.It blocks DNA replication in procaryotes
*2.blocks RNA synthesis in procaryotes
3.It binds to yeast sterols and interferes with membrane
4.It intereferes with vitamin (folic acid) synthesis.
5.It blocks the activity eucaryotic RNA polymerase.
13. Which of the following antibiotics inhibits bacterial protein synthesis?
1.penicillin
2.cephalosporin
*3.macrolides (e.g. erythromycin)
4.polymyxins
5.none of the above
14. Which of the following antimicrobial agents inhibits bacterial cell wall syn
thesis?
1.polymyxins
*2.cephalosporin
3.tetracycline
4. macrolides
5.none of the above
15.A 54-year-old woman suffering from influenza deteriorates and develops
shaking chills and a high fever. Physical examination is remarkable for dullness

to percussion at the left base and decreased breath sounds on the left.
Chest x-ray confirms the diagnosis of lobar pneumonia, presumed to be caused by
Streptococcus pneumoniae. The patient has no known drug allergies. Which of the
following
antibiotics would be most appropriate to treat the patient's condition?
*1.Cefotaxime
2.Chloramphenicol
3.Rifamycin
4.Penicillin
5.Vancomycin
16.Amoxicillin by contrast with benzylpenicillin is effective in
1. Staphylococcus infection
*2. Streptococcus viridans infection
3. Shigella infection
4. Salmonella infection
5. H. influenzae meningitis
17.The most long acting cephalosporin of 3-rd generation is
1.Cefalexin
2. Cefoperazon
*3. Ceftriaxon
4. Cephazolin
5. Cefaclor
18.The polypeptide antibiotic, which is active against gramnegative patogens onl
y
and used mainly topically, is
1. Vancomycin
2. Cliandamycin
*3. Polymyxin
4. doxycycline
5. Griseofulvin
19.Which of the following tetracyclines is safer for administration to persons
with decreased renal function
1.Minocycline
2.Tetracycline
3.Oxytetracycline
4.Chlortetracycline
*5.Doxycycline
@SULPHONAMIDES
1. The sulphonamides act as chemotherapeutical agents because they:
1. Intercalate with DNA
2. Inhibit RNA synthesis
3. Bind to the plasma membrane
*4. Are structural analogues of paraaminobenzoiñ acid
5. Chelate various metal ions
2. The fluorquinolones are contraindicated in children because they cause:
*1. Arthropathy
2. Agranulocytosis
3. Anemia
4. Thrombocytopenia
5. GIT ulceration
3. A 25-years-old patient addressed the doctor with complaints of redness and pa
in in the eyes
with purulent discharges. The ophthalmologist made the diagnosis of conjunctivit
is. Indicate
the sulfonamide which may be used for local application:
1. Biseptol
2. Streptocid
3. Aethazol
*4. Natrium sulfacyl
5. Sulfadimethoxine
4. Which group of chemotherapeutical action possesses next side effects:
nausea, vomiting, crystalluria, allergy, hepatitis
*1. Sulfonamides
2. Antitubercular drugs
3. Antibiotics
4. Antifungal drugs
5. Antiviral drugs
5. A 30 year old male is diagnosed to be HIV positive. His CD4+ count is 200 cel
ls/cm and his viral
load is 10,000 copies/mL. In addition to receiving antiviral therapy, which of t
he following
is indicated to protect him against pneumonia due to Pneumocystis jiro-veci?
1. Trimethoprim
2. Ciprofloxacin
*3. Co-trimoxazole
4. Clindamycin
6. A 26-year-old young man presents with the symp¬toms of gonorrhea. Because this
condition
is often associated with an infection due to Chlamydia trachomatis, which of the
following
quinolones would be the best choice for treating him?
1. Ciprofloxacin
2. Nalidixic acid
3. Norfloxacin
*4. Levofloxacin
7. In which one of the following infections is cipro¬floxacin ineffective?
1. Urinary tract infections due to a |3-lactamase-producing strain of klebs
iella
*2. Pneumonia due to Streptococcus pneumoniae
3. Exacerbation of chronic bronchitis due to Moraxella catarrhalis
4. Urinary tract infection due to Escherichia coli
5. Urinary tract infections due to Pseudomonas aeruginosa
8. Sulfonamides increase the risk of neonatal kernicterus because they
1. Diminish the production of plasma albumin.
2. Increase the turnover of red blood cells.
3. Inhibit the metabolism of bilirubin.
*4. Compete for bilirubin binding sites on plasma albumin.
5. Depress the bone marrow.
@ ANTIPROTOZOAL DRUGS
1. A patient of infectious department was diagnosed amoebiasis.
Indicate the antiamoebic drug.
*1. Metronidazole
2. Nystatin
3. Primaquine
4. Itraconazole
5. Solusurminum
2. Choose a drug for the treatment of Leishmaniasis.
1. Metronidazole
2. Itraconazole
3. Nystatin
*4. Solusurminum
5. Chloroquine
3. Nystatin exerts its effect by:
1. inhibition of folic acid synthesis
*2. impairment of cell membrane structure and function
3. inhibition of protein synthesis
4. interference with peptidoglycan synthesis
5. inhibition of nucleic acids synthesis
4. Antimicrobial and antiprotozoal effects of Metronidazole
are realized by following mechanism:
1. inhibition of protein synthesis
2. formation of complexes with sterols of cell membrane
3. competitive antagonism with PABA
*4. reduction of nitro-group and yielding reactive products
5. inhibition of nucleic acid synthesis
5. Name the drug for treatment of infection by nematodes:
1. Suramin
2. Primaquine
3. Metronidazole
*4. Mebendazole
5. Ketoconazole
6. Point oraly used antifungal antibiotic that is deposited
in keratin of nails and hair:
1. Ampicillin
2. Tetracycline
3. Nystatin
*4. Griseofulvin
5. Tamoxifen
7. Chronic candidiasis infections of the GIT and oral cavity
are treated with which agent in pill form:
1. Amphotericin B
*2. Nystatin
3. Miconazole
4. Fluconazole
5. Clotrimazole
8. Thiabendazole, a benzimidazole derivative, is an antihelminthic drug
used primarily to treat infections caused by:
1. Ascaris lumbricoides (roundworm)
2. N. Americanus (hookworm)
*3. Strongyloides
4. Enterobius vermicularis
5. Taenia saginata (flatworm)
9. The mechanism of action of chloroquine
in Plasmodium falciparum malaria is elimination of:
1. Secondary tissue schizonts
2. Exoerythrocytic schizonts
*3. Erythrocytic stage
4. Asexual forms
5. Sporozoites
10. A 60-year-old male with AIDS develops a systemic fungal infection
that is treated with fluconazole. What is the mechanism
of action of fluconazole?
*1. It inhibits ergosterol synthesis
2. It inhibits DNA synthesis
3. It inhibits peptidoglycan synthesis
4. It inhibits protein synthesis
11. A 35-year-old female complains of itching in the vulval area.
Hangingdrop examination of the urine reveals trichomonads.
What is the preferred treatment for trichomoniasis?
1. Doxycycline
2. Solusurmine
3. Sulfonamides
4. Chloridine
*5. Metronidazole
12. Tinnitus, dizziness, blurred vision, and headache are indicative
of toxicity to which one of the following antimalarial drugs?
1. Primaquine
*2. Quinine
3. Aminoachrichine
4. Chloroquine
5. Chloroquine
13. Following drug is effective against Trichomonas vaginalis.
1. Chloroquine
2. Primaquine
3. Cotrimaxazole
4. Furazolidone
*5. Metronidasole
14. Determine antifungal drug for treatment of candydamycosis
1. Griseofulvin
2. Cotrimaxazole
*3. Clotrimazole
4. Metronidasole
5. Furazolidone
15. A healthy man is|been| in dangerous on the disease
on a malaria district|region|. What from the noted preparations
must be appointed|set| with the purpose of private
chemical therapy against malaria.
1. Sulfalenum
2. Tetracyclinum
3. Metronidazolum
*4. Chloridinum
5. Biseptolum
16. For treatment of erythrocytic schizonticides for
malaria include the following drugs
*1. Chloroquine
2. Primaquine
3. Quinine
4. Chinidine
5. Mefloquine
17. Preparation has damage| influence on the red corpuscles forms
of malarial plasmodes, dysenteric amoeba. It is used for treatment
and prophylaxis of malaria, treatment to amoebiases and collagenosess.
Define this drug.
*1. Chingaminum
2. Aemethini hydrochloridum|
3. Tetracyclinum
4. Erythromycin
5. Quinine
18. Metronidasole can be used in the following except:
*1. Toxoplasmosis
2. Giardiasis
3. Leishmaniasis
4. Trichomoniasis
19. Antiprotozoal effect of metronidazole is realized by the
following mechanism
1. Inhibition of protein synthesi
*2. Reduction of nitro-groups and yielding reactive products
3. Competitive antagonism with PABA
4. Formation of complexes with sterols of cell membrane
5. Inhibition of nucleic acid synthesis
20. The following drug is effective for both intestinal
and extraintestinal amoebiasis
*1. Metronidasole
2. Chloroquine
3. Chiniofone
4. Emetine
5. Furazolidone
21. A 60-year-old male with AIDS develops a systemic fungal infection
that is treated with fluconazole. What is the mechanism of
action of fluconazole?
*1. It inhibits ergosterol synthesis
2. It inhibits dna synthesis
3. It inhibits peptidoglycan synthesis
4. It inhibits protein synthesis
22. Before the business trip|after| to the foreign countries
to the doctor|physician| with the purpose of the personal prophylaxis
of malaria appointed mean with shisontocydic action|act|.
What preparation did a specialist get|receives|?
*1. Chloridinum
2. Chiniofone
3. Doxicyclinum
4. Primaquine
5. Quinine
23. Name the drug for treatment of infecting by nematodes.
1. Aminoachrichine
2. Primaquine
3. Metronidazole
*4. Mebendazole
5. Praziquantel
24. A 22-year-old man complains of diarrhea and fatigue.
Examination of stool specimens shows binucleate organisms
with four flagellae. Which one of the following drugs
would be effective in treating this patient's infestation?
*1. Metronidazole
2. Quinidine
3. Pentamidine
4. Sulfadoxine
25. A 30-year-old male has had a heart transplant and is
being maintained on the immunosuppressant, cyclosporine.
He develops a Candida infection and is treated with ketoconazole.
Why is this poor therapy?
1. Ketoconazole is not effective against Candida.
2. Ketoconazole reacts with cyclosporine to inactivate it.
3. Ketoconazole has a potential for cardiotoxicity.
*4. Ketoconazole inhibits cytochrome P-450 enzymes
that inactivate cyclosporine.
5. Ketoconazole causes gynecomastia and decreased libido in the male.
26. A 36-year-old male of Lebanese ancestry is being
treated for vivax malaria. He experiences severe fatigue,
back pain, and darkened urine. Which one of the following
antimalarial drugs is most likely to have caused his symptoms?
1. Aminoachrichine
2. Chloridine
3. Chloroquine
4. Mebendazole
*5. Primaquine
@ Antivir, antimicobacterial
1. Impaired vision is an adverse effect of:
1. Carbenicillin
*2. Ethambutol
3. Rifampicin
4. Cotrimaxazole
5. Cycloserine
2. Isoniazid-induced liver damage:
1. occurs primarily in patients under 30 years of age
2. occurs with increased frequency in patients receiving
concomitant ethambutol therapy
*3. is probably due to the formation of a toxic hydrazine
metabolite that binds to liver protein
4. is frequently associated with allergic manifestations
such as eosinophilia, fever, and rash
5. All of the above
3. Tubercle bacilli exist in tuberculous patients in three pools extracellular,
intracellular, and necrotic caseum. The only drug which is bactericidal for
all three is:
*1. Isoniazid
2. Ethambutol
3. Pyrazinamide
4. Rifampicin
5. Streptomycin
4. Which of the following is a bactericidal antitubercular
drug that has an inhibitory action
on cell wall synthesis?
*1. Isoniazid
2. Rifampicin
3. Ethambutol
4. Streptomycin
5. Para-aminosalicylic acid
5. The route by which isoniazidum is usually administered is:
*1. Oral
2. Intramuscular injection of solution
3. Intramuscular injection of suspension
4. Subcutaneous injection of solution
5. Intravenous
6. An antitubercular agent which is associated with the
development of ocular toxicity is:
1. Rifampicin
*2. Ethambutol
3. Isoniazid
4. Streptomycin
5. Paraaminosalicylic acid
7. All of the following are considered primary drugs
in the current therapeutic approaches to
the treatment of tuberculosis EXCEPT:
1. streptomycin
*2. para-aminosalicylic acid
3. isoniazid
4. rifampicin
5. ethambutol
8. Failure of first-line drugs in the initial treatment
of apparent pulmonary tuberculosis
may have several causes including poor patient
compliance and atypical mycobacterial infections,
the latter usually requiring use of second-line drugs.
All of the following are first-line drugs EXCEPT:
1. streptomycin
2. rifampin
*3. ethionamide
4. ethambutol
5. isoniazid
9. A forty-year-old man has been on primary therapy for active pulmonary
tuberculosis for the past two months. At his regular clinic visit,
he complains of a "pins and needles" sensation in his feet.
You suspect that he might be deficient in which one of the following vitamins?
1. Ascorbic acid
2. Niacin
*3. Pyridoxine
4. Calcitriol
5. Folic acid
10. A 31-year-old white intravenous drug user was admitted
to the hospital with a four-week history of cough and fever.
A chest radiograph showed left upper lobe cavitary infiltrate.
Cultures of sputum yielded M. tuberculosis susceptible to all
antimycobacterial drugs. The patient received isoniazid,
rifampin, and pyrazinamide. The patient's sputum remained
culture-positive for the subsequent four months. Which one of the
following is the most likely cause of treatment failure?
1. False-positive cultures
2. Maladsorption of the medications
3. Concomitant infection with HIV
*4. Noncompliance by the patient
5. Drug resistance
11. A thirty-five-year-old male, formerly a heroin abuser, has been on methadone
maintenance for the last thirteen months. Two weeks ago, he had a positive PPD t
est, and
a chest radiograph showed evidence of right upper lobe infection. He was started
on standard antimycobacterial therapy. He has come to the emergency department
complaining of "withdrawal symptoms." Which of the following antimycobacterial
drugs is likely to have caused this patient's acute withdrawal reaction?
1. Ethambutol
2. Isoniazid
3. Pyrazinamide
*4. Rifampicin
5. Streptomycin
12. Amantadine, a synthetic antiviral agent used prophylactically against
influenza A2, is thought to act by
1. Preventing production of viral capsid protein
2. Preventing virion release
*3. Preventing penetration of the virus into the host cell
4. Preventing uncoating of viral DNA
5. Causing lysis of infected host cells by release of
intracellular lysosomal enzymes
13. Chills, fever, and muscle aches are common reactions to which one of
the following antiviral drugs?
*1. Interferon
2. Acyclovir
3. Ganciclovir
4. Ribavirin
14. In a dermatological and venerological dispenser comes a 30 years old woman w
ith
a diagnosis primary syphilis. Which drug is indicated in this case?
*1. Benzylpenicyllinum
2. Phenoximethipenicyllinum
3. Tetracyclinum
4. Laevomycetinum
5. Cefamezinum
15. What is the mechanism of action of zidovudine?
1. Inhibition of RNA synthesis
2. Inhibition of viral particle assembly
3. Inhibition of viral proteases
*4. Inhibition of nucleoside reverse transcriptase
5. Inhibition of nonnucleoside reverse transcriptase
16. A man during an epidemic of influenza appealed to the doctor
on an occasion a prophylaxis and early treatment of influenza.
What drug must be appointed in this case?
1. Methisazonum
2. Adamantane
3. Acyclovirum
*4. Interferonum
5. Bonaftonum
17. The epidemic of influenza is registered in the town. Which one from followin
g
drugs (nasal drops) should be prescribed to child to prophylaxis of influenza?
*1. Interferonum
2. Remantadinum
3. Ampicyllinum
4. Acyclovirum
5. Paracetamolum
18. An HIV-positive woman is diagnosed with CMV retinitis.
She has been on a HAART regimen containing zidovudine. Which of the
following anti-CMV drugs is likely to cause additive
myelosuppression with zidovudine?
1. Acyclovir
*2. Ganciclovir
3. Amantadine
4. Foscarnet
5. Ribavirin
19. An antitubercular agent which is associated with the development
of ototoxicity is:
1. Rifampicin
2. Ethambutol
3. Isoniazid
*4. Streptomycin
5. Paraaminosalicylic acid
20. Various drugs may induce vitamin deficient states as an
undesirable side effect. Vitamin B6 (pyridoxine) deficiency
may be related to taking:
*1. isoniazid
2. ethambutol
3. pyrazinamide
4. rifampicin
5. streptomycin
21. Which of the following drugs is most closely associated
with the development of renal and ototoxicity?
*1. kanamycin
2. penicillin G
3. tetracycline
4. isoniazid
5. ethambutol
22. Herpetic infection can be treated by local use of
the following drugs except:
*1. Interferon
2. Acyclovir
3. Vidarabine
4. Idoxuridine
5. Trifluridine
23. Determine the Acyclovir mechanism of action:
1. Inhibits of virus penetration of host cell
*2. Inhibits of DNA polymerase
3. Immunomodulation
4. Inhibits of RNA translocase
5. Immunosuppression
@Anticancer drugs
1. A nucleophilic attack on deoxyribonucleic acid (DNA)
that causes the disruption of base pairing occurs
as a result of the administration of:
*1. Cyclophosphamide
2. 5-FU
3. Methotrexate
4. Prednisone
5. Tamoxifen
2. Which of the following is a chemotherapeutic drug
that acts through alkylation?
1. Tamoxifen
2. Methotrexate
*3. Cyclophosphamide
4. 5-FU
5. Doxorubicin
3. Binding to the enzyme dihydrofolate reductase
is the mechanism of action for:
1. Cyclophosphamide
*2. Methotrexate
3. Tamoxifen
4. 5-FU
5. Doxorubicin
4. A 34-year-old male with Hodgkin s disease
is treated with the vinblastine.
What is the mechanism of action of vinblastine?
1. Scission of DNA strands
2. Inhibition of dihydrofolate reductase
3. Inhibition of enzymes involved in purine metabolism
*4. Prevention of assembly of tubulin dimers into microtubules
5. Inhibition of topoisomerase
5. A 41-year-old female is treated for
endometrial cancer with tamoxifen.
Of the following, how is tamoxifen classified?
1. An alkylating agent
2. An antimetabolite
3. A plant alkaloid
4. An antibiotic
*5. A hormone antagonist
6. A 35-year-old female is being treated
for cervical cancer with cisplatin.
Of the following, how is cisplatin classified?
*1. An alkylating agent
2. An antimetabolite
3. A plant alkaloid
4. An antibiotic
5. A hormonal agent
7. A 45-year-old female treated for ovarian cancer
develops difficulty in hearing. Which of the
following agents most likely caused his defect?
1. Vincristine
2. Doxorubicin
3. Bleomycin
4. 5-FU
*5. Cisplatin
8. Determine the anticancer drug that act as purine antagonist.
1. Vincristine
2. Actinomycin
3. Phosphestrol
*4. Mercaptopurine
5. L-asparaginase
9. To the patient, that suffers on a acute leucosis
appointed antineoplastic drug with antimetabolic action,
antagonist of folic| acid. What drug appointed?
1. Ftorupacylum
*2. Metotrexat
3. Mielosanum
4. Mercaptopurinum
5. Sinestrolum
10. In basis|foundation| of antitumoral action of alkalizing
facilities their influence lies on cellular to DNA and RNA,
which in this connection|as a result| lose a capacity
for duplication. What most early complications
from application of these drugs can arise up in connection
with the indicated|specified| mechanism of their antitumoral action?
*1. Blood disturbances
2. Cardiological pathology
3. Nephrotoxicity
4. Neurotoxicity
11. The sick 60 years carried a mammectomy.
After the course of radial therapy a doctor appointed
synthetic drug with non-steroidal structure,
that removes stimulant influence of estrogens
on tumor growth. What preparation was appointed?
1. Rubomycinum
2. Cysplastinum|
3. Diethylstilbestrolum
4. Phosfestrolum|
*5. Tamoxifen
12. Antitumoral drug| that behaves to the group
of antimetabolites is used for treatment of acute leucosis
at children|kids, and also at adults for treatment of solid tumours.
Define this drug.
1. Sarcolisinum
2. Colchaminum
*3. Metotrexat
4. Rubomycinum
5. Prednisolonum
13. A patient with colonic cancer is being
treated with 5-fluorouracil as well as leucovorin.
The rationale for administering the coenzyme
depends on its being essential for:
1. Conversion of 5-fluorouracil to fluorodeoxyuridylic acid (FdUMP).
*2. Protection against the anemia caused by 5-fluorouracil treatment.
3. The inhibition of thymidylate synthase by FdUMP.
4. Prolongation of the antitumor effect of 5-fluorouracil.
14. Neutropenia develops in a patient undergoing
cancer chemotherapy. Administration of which one
of the following agents would accelerate recovery
of neutrophil counts?
*1. Leucovorin
2. Filgrastim
3. Prednisone
4. Vitamin B12
15. Hydration and/or diuresis can prevent the renal toxicity associated with:
1. Cisplatin
*2. Chlorambucil
3. Tamoxifen
4. Gemcitabine
5. Methotrexate
@POISONING
1. Convulsions caused by drug poisoning are most
commonly associated with:
1. Phenobarbital
*2. Diazepam
3. Strychnine
4. Chlorpromazine
5. Phenytoin
2. Alkalinization of the urine with sodium bicarbonate is
useful in the treatment of poisoning with:
*1. Aspirin (acetylsalicylic acid)
2. Amphetamine
3. Morphine
4. Phencyclidine
5. Cocaine
3. A 50-year-old male who is chronic alcoholic ingests methanol.
Which of the following findings is associated
with acute methanol ingestion?
1. Metabolic alkalosis
2. Delirium tremens
3. An atrioventricular conduction defect
*4. Blurred vision
5. Tachypnia
4. A 20-year-old male ingest iron pills by accident. He develops
severe abdominal pain and bloody vomitting.
Which of the following might be administered to this patient?
1. Dimercaprol
*2. Deferoxamine
3. Penicillamine
4. Na2EDTA
5. Activated charcoal
5. A male patient is brought to the emergency department (ED)
following ingestion of an unknown substance.
He is found to have an elevated temperature, hot and flushed skin,
dilated pupils, and tachycardia. Of the
following, which would most likely cause these findings?
1. Propranolol
2. Proserine
3. Prazosin
4. Guanethidine
*5. Atropine
6. A 25-year-old male is seen in the emergency department (ED).
He is disoriented but developed nausea,
vomiting, abdominal pain, and diarrhea since he took
too many pain pills. Before he can tell you more, he loses
consciousness. Liver function tests are abnormal.
In addition to gastric lavage, what is the appropriate treatment?
1. Naloxone
2. Deferoxamine
3.N-acetylcysteine
*4. Proserine
5. Alloxime
7. A 72-year-old female with a long history of anxiety
treated with diazepam decides to triple her dose because
of increasing fearfulness about environmental noises.
after several days,she became extremely lethargic and
nonresponsive. On examination, she is found to be stuporous
and have diminished reaction to pain and decreased
reflexes. Her respiratory rate is 8 breaths per minute (BPM),
and she has shallow respirations. Which antidote
could be given to reverse these findings?
1. Naltrexone
2. Physostigmine
3. Alloxime
*4. Flumazenil
8. A 38-year-old man who poisoned himself with mercury
dichloride was taken to the admission room in grave
condition. What antidote should be immediately introduced?
1. Isonitrosine
2. Nalorphine
3. Dipiroxim
4. Atropine
*5. Unithiol
9. Drug for treatment of acute iron poisoning:
1. Alloxime
*2. Deferoxamine
3. Atropine
4. Flumazenil
5. Ferrous succinate

También podría gustarte